Download Назва наукового напрямку (модуля): Семестр: 4 Модуль 4 Цикл

Survey
yes no Was this document useful for you?
   Thank you for your participation!

* Your assessment is very important for improving the workof artificial intelligence, which forms the content of this project

Document related concepts

Heart failure wikipedia , lookup

Coronary artery disease wikipedia , lookup

Hypertrophic cardiomyopathy wikipedia , lookup

Mitral insufficiency wikipedia , lookup

Artificial heart valve wikipedia , lookup

Myocardial infarction wikipedia , lookup

Cardiac surgery wikipedia , lookup

Electrocardiography wikipedia , lookup

Antihypertensive drug wikipedia , lookup

Dextro-Transposition of the great arteries wikipedia , lookup

Quantium Medical Cardiac Output wikipedia , lookup

Transcript
Назва наукового напрямку (модуля):
Семестр: 4
Модуль 4 Цикл Physiology of blood circulatory system(2c.-med.Sem.Contr)
Опис:
Перелік питань:
1.
A. *
B.
C.
D.
E.
2.
A. *
B.
C.
D.
E.
3.
A.
B.
C.
D.
E. *
4.
A.
B.
C. *
D.
E.
5.
A.
B.
C.
D. *
E.
6.
A.
B.
C.
D. *
E.
On the experiment on a dog, it was set that a law of Starling for a heart represents:
Ability of heart to increase the power of contractions during the increase of filling of his chambers
with blood.
Correlation of consumption of oxygen by myocardium with work, that is being done
Correlation of volume of right atrium with the frequency of cardiac contractions
Correlation of the cardiac ejection with the peripheral resistance
Change of tonus of veins and skeletal muscles
Humans have two circles of blood circulation. From what part of the heart does the pulmonary circuit
starts?
From the right ventricle
From the left ventricle
From the right atrium
From the left atrium
From the sinus node
An important feature of myocardium is the presence of the plateau phase, that is caused by the
change of permeability of membranes for definite ions. What ion is it , and what is it’s way through
the membrane ?
Exit of Na+ ions
Entrance of Na+ ions
Exit of K+ ions
Exit of Ca2+ ions
Entrance of Ca2+ ions
It is known, that a QRS complex represents the process of distribution of depolarization wave on
right and left ventricles. Excitation of what area of ventricles is characterized by the Q indent?
lateral walls of heart
Superior and middle third of an interventricular septum
Middle and inferior third of an interventricular septum
Superior third of an interventricular septum
Basal departments of the heart
It is known, that a QRS complex represents the process of distribution of depolarization wave on
right and left ventricles. Excitation of what area of ventricles is characterized by the S indent?
Superior and middle third of an interventricular septum
Middle and inferior third of an interventricular septum
the apex of the heart
Basal departments of the heart and the superior third of interventricular septum
lateral walls of heart
A 26-year-old man is diagnosed with a sinus bradycardia. The frequency of cardiac contractions is
about 52 heartbeats per minute. Which one of the following properties is damaged?
Conductivity
Excitability
Refractivity
Automatism
Refractivity, excitability
7.
A.
B.
C.
D. *
E.
8.
A. *
B.
C.
D.
E.
9.
A. *
B.
C.
D.
E.
10.
A.
B.
C.
D.
E. *
11.
A. *
B.
C.
D.
E.
12.
A.
B.
C.
D. *
E.
13.
A.
During the lesson, students removed the heart of a frog and placed it in isotonic solution, where it
continued to contract. What property causes the isolated heart to contract?
Conductivity
Excitability
Refractivity
Automatism
Excitability, refractivity
After imposing the ІІ ligature of Stanius, students established that the frequency of atrium
contractions is 55 /minute, and the frequency of ventricle contraction - 40 /minute. It is explained by
the gradient of automatism of different pacemakers. What is it caused by?
Different permeability of membranes for Na+
Different permeability of membranes for K+
Different content of Ca2+
Different activity of Na+, K+-pump
All of the listed factors
It is known, that a cardiac muscle can contract only in the mode of single contraction. It is explained
by the long duration of a period of refractivity. What causes such duration of this period?
Inactivation of Na+-channels
Activation of Na+- channels
Inactivation of K+- channels
Activation of K+- channels
Inactivation of Ca2+- channels
An important feature of myocardium is the presence of the plateau phase, that is caused by the
change of permeability of membranes for definite ions. What ion is it , and what is it’s way through
the membrane ?
Exit of Na+ ions
Entrance of Na+ ions
Exit of K+ ions
Exit of Ca2+ ions
Entrance of Ca2+ ions
In what space is the difference of potentials registered during the ECG recording in standart lead?
Frontal
Sagittal
Horizontal
Vertical
Frontal and sagittal
In what space is the difference of potentials registered during the ECG recording in pectoral leads?
Frontal
Sagittal
Vertical
Horizontal
Frontal and sagittal
During registration of the ECG in V2 lead, an active electrode was set on the patient in III intercostal
region to the left of breastbone(sternum),that is not correct. Where is an electrode supposed to be
placed in this case?
In III intercostal region to the right of breastbone
B.
C. *
D.
E.
14.
A.
B. *
C.
D.
E.
15.
A.
B.
C.
D.
E. *
16.
A.
B. *
C.
D.
E.
17.
A. *
B.
C.
D.
E.
18.
A.
B.
C. *
D.
E.
19.
A.
B.
C.
D. *
E.
20.
In IV intercostal region to the right of breastbone
In IV intercostal region on the left of breastbone
In V intercostal region on the left medio-clavicular line
In V intercostal region on the left medium axillary line
During the registration of an electrogram of a single muscular fibre, the vector of dipole is directed to
the side of the negative electrode of lead. What will be the direction of the wave on an electrogram in
this case ?
Positive
Negative
Diphasic
Izoline is registered
No registered
It is known, that a QRS complex represents the process of distribution of depolarization wave on
right and left ventricles. Excitation of what area of ventricles is characterized by the R indent?
Superior and middle third of an interventricular septum
Middle and inferior third of an interventricular septum
Superior third of an interventricular septum
Basal departments of the heart
lateral walls of heart
During the examination, the rise of systolic pressure was found in a 45 years old patient. Exceeding
of what level of pressure testifies it?
129 mm Hg
139 mm Hg
149 mm Hg
159 mm Hg
130 mm Hg
What direction does the repolarization wave in the myocardium of ventricles have?
From epicardium to endocardium
From endocardium to epicardium
Along endocardial layers
Along epicardial layers
From the apex to the lateral walls of the heart
A 48 years old patient suffers the bronchial asthma. In the II standard lead, the duration of the P wave
makes 0.14 seconds. What is the normal duration of this wave?
Up to 0,15 sec
Up to 0,03 sec
Up to 0,1 sec
Up to 0,12 sec
Up to 0,24 sec
What indent represents the process of repolarization of the ventricles?
Indent R
Indent Q
Indent S
Indent T
Cannot be registered
In what space is the difference of potentials registered during the ECG recording in increased leads?
A.
B.
C. *
D.
E.
21.
A.
B. *
C.
D.
E.
22.
A.
B.
C. *
D.
E.
23.
A.
B. *
C.
D.
E.
24.
A.
B. *
C.
D.
E.
25.
A.
B.
C.
D. *
E.
26.
A.
B.
C.
Horizontal
Sagittal
Frontal
Vertical
Frontal and sagittal
During registration of the ECG in V1 lead, an active electrode was set on the patient in II intercostal
region to the left of breastbone(sternum),that is not correct. Where is an electrode supposed to be
placed in this case?
In II intercostal region to the right of breastbone
In IV intercostal region to the right of breastbone
In IV intercostal region on the left of breastbone
In V intercostal region on the left medio-clavicular line
In V intercostal region on the left anterior axillary line
A 60 year old patient M. had an acute heart attack. The cardiogram in I, II standard and V4-V6 chest
leads show the deep and wide wave Q. What is the normal amplitude of the Q wave?
Up to 5 % of the amplitude of R wave
Up to 15 % of the amplitude of R wave
Up to 25 % of the amplitude of R wave
Up to 15 % of the amplitude of S wave
Up to 25 % of the amplitude of S wave
How does the work of heart of patients with the diabetes mellitus changes?
Heart beat rate will be increased
Heart beat rate will decrease
Power of cardiac contractions will be increased
Speed of conducting of excitation will decrease
Speed of conducting of excitation will be increased
A 62 years old patient has pathology of the coronary blood circulation. The I, II standard and the
V3-V4 leads show the negative T wave. Normally, it has to be negative only in one lead. What is this
lead?
aVF
aVR
aVL
ІІІ standard
V1.
Doctors have identified the maximal amplitude of the R wave in V6 lead of the 38 years old man. In
what chest lead its amplitude is the greatest?
V1
V2
V3
V4
V5
During registration of the ECG in V4 lead, an active electrode was set on the patient in IV intercostal
region on the left parasternal line, that is not correct. Where is an electrode supposed to be placed in
this case
In V intercostal region on the left medium axillary line
In IV intercostal region to the left of breastbone
In IV intercostal region to the right of breastbone
D.
E. *
27.
A. *
B.
C.
D.
E.
28.
A.
B.
C. *
D.
E.
29.
A.
B.
C.
D. *
E.
30.
A.
B. *
C.
D.
E.
31.
A.
B.
C. *
D.
E.
32.
A.
B. *
C.
D.
E.
In V intercostal region on the left anterior axillary line
In V intercostal region on the left medio-clavicular line
During registration of the ECG in V6 lead, an active electrode was set on the patient in the point
where the left V cost (rib) attaches to the breastbone(sternum), that is not correct. Where is an
electrode supposed to be placed in this case?
In V intercostal region on the left medium axillary line
In V intercostal region on the left anterior axillary line
In IV intercostal region on the left of breastbone
In V intercostal region on the left medio-clavicular line
In the place of attachment of the right V rib to the breastbone
Before registering an ECG, the calibration signal must be recorded first. What is the value of the
standard calibration signal in the electrocardiographs?
0,1 mV
0,01 mV
1,0 mV
5 mV
10 mV
In order to find out the way of electric axis of the heart one must define the algebraic sum of waves in
certain ECG leads. What waves are they and in what leads their sums are found?
QRST in ІІ and ІІІ standard leads
QRS in ІІ and ІІІ standard leads
PQRS in I and ІІІ standard leads
QRS in I and ІІІ standard leads
QRST in I and ІІІ standard leads
While finding out the direction of the electric axis of the heart, it was determined that angle α equals
+25°. What position of the electric axis of the heart makes such a result?
Vertical
Horizontal
Intermediate
Decline to the left
Decline to the right
While finding out the direction of the electric axis of the heart, it was determined that angle α equals
+67°. What position of the electric axis of the heart makes such a result?
Vertical
Horizontal
Intermediate
Decline to the left
Decline to the right
What formula needs to be used in order to estimate the frequency of cardiac contractions using the
ECG?
Frequency of cardiac contractions = duration of R-R interval in seconds : 60 s
Frequency of cardiac contractions = 60 s.: duration of R-R interval in seconds
Frequency of cardiac contractions = 60 s · duration of R-R interval in seconds
Frequency of cardiac contractions = duration of P-R interval in seconds : 60 s.
Frequency of cardiac contractions = 60 s. : duration of P-R interval in seconds
33.
A.
B.
C. *
D.
E.
34.
A.
B. *
C.
D.
E.
35.
A. *
B.
C.
D.
E.
36.
A.
B.
C.
D. *
E.
37.
A. *
B.
C.
D.
E.
38.
A.
B.
C. *
D.
E.
39.
A.
B. *
At a 50 years old patient the duration of the QRS complex in II standard lead makes 0,11 sec. What
is the normal duration of this complex in standard leads?
Up to 0,03 sec
Up to 0,12 sec
0,06-0,09 sec
More than 0,1 sec
More than 0,03 sec
While examining the 27 years old sportsman, the U wave was registered on the ECG in II standard
lead. When is this indent usually registered?
At tachycardia
At bradycardia
At hypertrophy of the right ventricle
At hypertrophy of the atriums
At hypertrophy of ventricles
Doctors found the maximal amplitude of the S wave in V6 lead in 45 years old man. In what chest
lead the amplitude of the S wave is the greatest in a norm?
V1
V2
V3
V4
V5
A 39 years old patient suffers the arterial hypertension. The cardiogram in I standard lead shows the
amplitude of R wave of 27 mm. What is the normal amplitude of this wave in standard lead?
Up to 10 mm
Up to 15 mm
Up to 5 mm
Up to 20 mm
Up to 25 mm
Doctors have identified the deviation of the S-T segment in comparement to isoline, that made 2 mm
lower in V3-V4 leads. What is the normal position of this segment in chest lead?
Up to 0,5 mm downward
Up to 1 mm downward
Up to 2 mm downward
Up to 1 mm upward
Up to 5 mm downward
A 48 years old patient has the difference of duration of the R-R intervals 0.18 sec. What is the normal
difference of duration of the R-R intervals in normal heart beat rhythm?
Up to 0,12 sec
Up to 0,2 sec
Up to 0,1 sec
Up to 0,01 sec
Up to 0,06 sec
While finding out the direction of the electric axis of the heart, it was determined that angle α equals
+75°. What position of the electric axis of the heart makes such a result?
Intermediate
Vertical
C.
D.
E.
40.
A.
B.
C. *
D.
E.
41.
A.
B.
C.
D.
E. *
42.
A. *
B.
C.
D.
E.
43.
A.
B.
C.
D. *
E.
44.
A.
B.
C. *
D.
E.
45.
A.
B.
C. *
D.
E.
46.
Horizontal
Decline to the left
Decline to the right
During the analysis of ECG of the 49 years old patient C., violation of rhythm is set with localization
of the driver of rhythm in the left atrium. What signs indicate on localization of rhythm in sino-atrial
node? one of the signs
Neither
Presence of the positive R wave
Presence of the positive P wave before every QRST complex
Presence of the positive T wave before every QRS complex
Presence of the positive PQRST waves
It is known that the Q-T interval is called the electric systole of the heart. What process represents
this interval?
Depolarization of the atriums
Depolarization of the ventricles
Depolarization and repolarization of the atriums and ventricles
Depolarization and repolarization of the atriums
Depolarization and repolarization of the ventricles
The ECG shows the considerable increasement of the P-Q segment. Where is the conducting of
excitation slowed down?
In the atrio-ventricular node
Atriumm
Fascicle of Giss
Purkinje’s fibres
Ventricles
On the lesson, through the isolated heart of frog the 3 % solution of CaCl2 was passed by. How will
the heart work change?
power of cardiac contractions will decrease, heart beat rate will increase
speed of conducting of excitation will increase
heart will stop in a diastole
heart will stop in a systole
decrease of excitability
The 49 years old man has the equal amplitude of S and R waves in V4 taking. In what chest lead their
amplitude is equal in a norm?
V1
V2
V3
V4
V5
On echocardiography, a stroke volume of the 30 years old man made 75 ml. What is the normal
stroke volume for men?
60-80 ml
60-90 ml
70-90 ml
80-120 ml
50-70 ml
The Boudichi phenomenon sets the dependence of power of cardiac contractions on:
A. *
B.
C.
D.
E.
47.
A.
B.
C. *
D.
E.
48.
A.
B. *
C.
D.
E.
49.
A.
B.
C.
D.
E. *
50.
A.
B.
C.
D. *
E.
51.
A.
B.
C.
D.
E. *
52.
A.
B.
C. *
D.
Heart beat rate
degree of dilatation of cardiomyocites
pressure in an aorta
volume of ventricles
speed of conducting of excitation
During the examination of a 68 years old patient D., the end-systolic capacity made 65 ml. What is
the normal value of this index?
48-64 ml
38-50 %
38-50 ml
70-90 ml
60-90 ml
It is known that the pressure in left ventricle in systole equals about 110 mm Hg. What is the normal
pressure in left ventricle in diastole
25 mm Hg
0 mm Hg
10 mm Hg
15 mm Hg
3-8 mm Hg
It is known that the pressure in left atrium in diastole equals about 0 mm Hg. What is the normal
pressure in left atrium in systole?
0 mm Hg
25 mm Hg
10 mm Hg
15 mm Hg
3-8 mm Hg
On the echocardiography of the 56 years old man, doctors estimated the decrease of ejection fraction
of the left ventricle. What is the normal value of this index?
60-90 ml
60-90 %
54-70 %
54-64 %
38-50 ml
While examining the patient, doctors had to find the value of cardiac ejection fraction What method
can be used?
Sphygmography
X-ray examination
ECG
FCG
Echocardiography
Doctors have prescribed the β-adrenostimulators to a patient N. with bronchial asthma. How will the
work of heart after the reception of these medications change?
duration of the diastole will be increased
duration of the systole will be increased
duration of the diastole will decrease
duration of the systole will decrease
E.
53.
A.
B.
C.
D.
E. *
54.
A. *
B.
C.
D.
E.
55.
A.
B.
C.
D.
E. *
56.
A.
B. *
C.
D.
E.
57.
A. *
B.
C.
D.
E.
58.
A.
B.
C.
D. *
E.
59.
A.
B. *
delay time of conducting of excitation in atrio-ventricular node will be increased
The second diastolic tone of heart appears with:
Transition of blood from ventricles to large vessels
Filling of ventricles with blood
Filling of atriums with blood
Reversed motion of blood and closing of semilunar valves of aorta and pulmonary trunk
All of the named factors
On the lesson, through the isolated heart of frog the solution was passed by. It caused the increase of
heart beat rate. What solution was used?
Adrenalin
1 % solution of NaCl
Potassium chloride
Acetylcholine
NaHCO3
Fourth tone on fonocardiogram is conditioned by:
Filling of atriums
Ventricle contractions
Injection of blood to the pulmonary trunk
Injection of blood to the aorta
Atrium contractions and filling of ventricles with blood
16 years old girl, while rapid transition from horizontal position in to vertical position lost
consciousness. What of the following caused that:
Increase of the venous return of blood to the heart
Decrease of the venous return of blood to the heart
Increase of the frequency of cardiac contractions
Decrease of the frequency of cardiac contractions
Increase of arterial pressure
At a patient with a syndrome of Itsenco-Coushing the production of glucocorticoids was
considerably increased. How will the work of heart change?
heart beat rate will increase
heart beat rate will decrease
speed of conducting of excitation in atrio-ventricular node will decrease
power of cardiac contractions will decrease
the work of heart will not change
A 30 years old patient who had a stenosis (decrese of diameter) of the left atrio-ventricular opening
was diagnosed with the pulmonary hypertension. The systolic pressure in pulmonary artery made 30
mm Hg. What is the normal value of this index?
5-10 mm Hg
10-15 mm Hg
15-20 mm Hg
20-25 mm Hg
25-30 mm Hg
It is known that the pressure in right ventricle in systole equals about 20 mm Hg. What is the normal
pressure in right ventricle in diastole?
15 mm Hg
0 mm Hg
C.
D.
E.
60.
A. *
B.
C.
D.
E.
61.
A.
B. *
C.
D.
E.
62.
A.
B.
C.
D. *
E.
63.
A. *
B.
C.
D.
E.
64.
A.
B. *
C.
D.
E.
65.
A. *
B.
C.
D.
E.
10 mm Hg
20 mm Hg
3-8 mm Hg
On the lesson, students placed the isolated heart of frog in solution. No changes in heart work
happened. In what solution was the heart placed?
1 % solution of NaCl.
3 % solution of HCl
1 % solution of CaCl2
3 % solution of KCl
1 % solution of NaHCO3
It is known that the pressure in right atrium in systole equals about 5 mm Hg. What is the normal
pressure in right atrium in diastole?
25 mm Hg
0 mm Hg
10 mm Hg
15 mm Hg
3-8 mm Hg
During the preparation of a patient to the operation on a heart, measuring of pressure in chambers of
the heart was determined. In one of them pressure made 120 mm Hg (16,0 kPa). Name this chamber
of the heart.
left atrium
right ventricle
right atrium
left ventricle
venous sinus
Patients with heart pathologies often have the increased blood level of a hormone which decreases
the reabsorbsion of Na+ and water in tubules, and is produced in a heart. What hormone has such
action?sodium uretic hormone
Atrial
Rennin
Aldosteron
Vasopressin
Adrenalin
The first tone, formed by a right ventricle, is auscultated:
In second intercostal region on the left of breastbone
On the breastbone, where a processus xiphoid joins to it
Above semilunar valves
Above atrio-ventricular valves
In the projection of the apex of heart
Doctors during the inspection defined the presence of tachycardia in the state of rest. What can be its
reason?
Increased function of thyroid gland
Increased function of cortex of adrenal glands
Increased permeability of Cl-canals of myocardium
Increased tonus of the parasympathic system
Change of all listed factors
66.
A.
B. *
C.
D.
E.
67.
A.
B. *
C.
D.
E.
68.
A.
B. *
C.
D.
E.
69.
A.
B.
C.
D. *
E.
70.
A.
B. *
C.
D.
E.
71.
A.
B.
C.
D. *
E.
72.
A.
B.
C. *
D.
During the class, students conducted an orthostatic test. Thus there were the changes of
hemodynamics. What compensatory mechanisms provide the support of hemodynamics at this test?
Increase of arterial pressure
Increase of the frequency of cardiac contractions
Decrease of arterial pressure
Reduction of the frequency of cardiac contractions
Increase of the power of cardiac contractions
At a patient B., with the increased function of thyroid gland, the increase of heart beat rate is set.
What is the mechanism of the thyroid hormone influence on the heart?
decrease of quantity of B-adrenoreceptors
increase of quantity of B-adrenoreceptors
increase of quantity of N-cholinoreceptors
decrease of quantity of alfa-adrenoreceptors
increase of quantity of M- cholinoreceptors
What characterizes a protosphygmic interval?
Opening of semilunar valves
Closing of semilunar valves
Opening of atrioventricular valves
Closing of the bicuspid valve
Closing of atrioventricular valves
Doctors have intravenously ejected the solution of calcium chloride at a 45 years old male. What
effects will it stimulate?
Positive chronotropic and negative inotropic
Negative chronotropic and inotropic
Negative bathmotropic and inotropic
Positive chronotropic and inotropic
Work of heart won’t changes
A 56 years old patient K, with arterial hypertension, has used diuretics for a long time, that resulted
in violation of cardiac rhythm. By the change of concentration of what ion is this state conditioned?
increase of K+
decrease of K+
increase of Na+
decrease of Na+
decrease of Ca2+
An effect of Anrep sets the dependence of power of contractions of ventricle myocardium on:
Volume of ventricles
Tonus of the sympathetic system
Tonus of the parasympathetic system
Pressure in an aorta
Degree of dilatation of muscular fibers
In what conditions does the atrial natriuretic peptide excretion increases (by the incretory auricle
atrium cells)?
with the increase of concentration of K+
with the decrease of concentration of K+
with the increase of volume of circulatory blood
with the decrease of the volume of circulatory blood
E.
73.
A.
B.
C.
D. *
E.
74.
A.
B.
C.
D. *
E.
75.
A. *
B.
C.
D.
E.
76.
A. *
B.
C.
D.
E.
77.
A. *
B.
C.
D.
E.
78.
A. *
B.
C.
D.
E.
79.
A. *
B.
with the increase of level of Ca2+.
On the lesson, the students placed the isolated heart of frog in a solution. The heart stop in a diastole
happened. What solution was heart placed in?
1 % solution of NaCl.
3 % solution of NaCl
1 % solution of CaCl2
3 % solution of KCl
1 % solution of NaHCO3
On the lesson, through the isolated heart of frog the solution was passed by. It caused the decrease of
heart beat rate. What solution was used?
Adrenalin
Histamine
Calcium chloride
Acetylcholine
1 % NaHCO3.
At a patient with a syndrome of Cone the production of aldosteron was considerably increased. How
will the work of heart change?
heart beat rate will increase
heart beat rate will decrease
speed of conducting of excitation in atrio-ventricular node will decrease
power of cardiac contractions will decrease
the work of heart will not change
It is known, that the heart beat rate decreases under the action of acetylcholine. What mechanism
predetermines it?
duration of phase of spontaneous diastolic depolarization is increased
duration of phase of spontaneous diastolic depolarization decreases
speed of conducting of excitation to atrio-ventricular node is increased
duration of action potential is increased
duration of action potential is increased
Doctors have prescribed the beta-adrenoblockators to a patient K. with arterial hypertension. How
will the work of heart after the reception of these medications change?
duration of the diastole will be increased
duration of the systole will be increased
duration of the diastole will decrease
duration of the systole will decrease
delay time of conducting of excitation in atrio-ventricular node will be decreased
During the experiment on an animal, the right vagus nerve was cut. What changes would be
observed?
Increase of heart beat rate
Increase of power of cardiac contractions
Increase of speed of conducting of the excitation in an atrio-ventricular node
Decrease of heart beat rate
Decrease of power of cardiac contractions
On the lesson, students inflicted the strong irritation of vagus nerve of a rat, that caused the heart to
stop. What is the nature of this phenomenon?
activation of K+ - channels of the cells of sinoatrial node
activation of Na+ - channels of the cells of sinoatrial node
C.
D.
E.
80.
A.
B.
C.
D.
E. *
81.
A.
B.
C. *
D.
E.
82.
A.
B. *
C.
D.
E.
83.
A.
B.
C.
D.
E. *
84.
A.
B.
C.
D. *
E.
85.
A.
B. *
C.
D.
E.
inactivation of the Na+ - channels of the cells of sinoatrial node
activation of the Ca2+ - channels of the cells of sinoatrial node
inactivation of the Ca2+ - channels of the cells of sinoatrial node
It is known, that in emergency therapy for the removal of the attack of paroxysmal tachycardia,
doctors use the reflex, that is inducted by pressing on the eyeballs. How is this reflex called?
Hering’s
Holt’s
Bekhterev’s
Helmholt’s
Ashner’s
On practical the students during the experiment conducted electro-stimulation of the left vagus nerve.
What effect here will be observed?
Positive inotropic effect
Positive chronotropic effect
Negative inotropic effect
Positive dromotropic effect
No bathmotropic effect
On the lesson, students inducted the reflex by inflicting the strikes on the anterior abdominal wall of
frog. How is this reflex called?
Hering’s
Holt’s
Bekhterev’s
Valsalve’s
Ashner’s
It is known, that during the hard physical work, the heart beat rate is increasing. What receptors
conduct the afferent information in this case?
Mechanoreceptors of vessels
Chemoreceptors of muscles
Chemoreceptors of carotid sinus
Mechanoreceptors of the right atrium
Proprioreceptors
During the acute experiment on an animal, a part of spinal cord was destroyed. How will the heart
activity change in such conditions?
Increase of heart beat rate
Increase of power of cardiac contractions
Increase of conducting speed of excitation in atrio-ventricular node
Decrease of heart beat rate
Decrease of end-diastolic volume
On the lesson, students observed the respiratory arrhythmia. Thus on the monitor of cardioscope was
evidently, that at inhalation the heart beat rate was increasing, and at exhalation - decreasing. How
will the stroke volume (SV) change at the inhalation and exhalation?
At inhalation will increase, at exhalation - will decrease
At inhalation will decrease, at exhalation - will increase
Will increase at inhalation, at exhalation will not change
Will increase at exhalation, at inhalation will not change
Changes of stroke volume at inhalation and exhalation would not be observed
86.
A.
B.
C. *
D.
E.
87.
A. *
B.
C.
D.
E.
88.
A. *
B.
C.
D.
E.
89.
A.
B.
C.
D.
E. *
90.
A.
B.
C.
D.
E. *
91.
A. *
B.
C.
D.
E.
92.
A.
B.
C.
It is known, that the frequency of cardiac contractions decreases during a sleep. What mechanism
causes such changes?
Inhibition of sensory areas of the cerebral cortex
Inhibition of motor areas of the cerebral cortex
Increase of tonus of n.vagus
Increase of tonus of the sympathetic nervous system
Inhibition of neurons of middle hypothalamus
The dicrotic wave on sphygmogram appears as a result of:
Push of blood into closed semilunar valves of aorta
Push of blood into closed semilunar valves of pulmonary trunk
Transition of blood from ventricles in an aorta
Push of blood into closed mitral valves
Push of blood into closed tricuspid valves
One of important haemodynamic indexes is the linear speed of blood-flow. What factor does this
index depend on mainly?
Total area of cross-section of vessels
Length of vessel
Viscosity of blood
Size of stroke volume
Gradient of pressure
During the prophylactic examination of a 25 years old man, arterial pressure was measured on hands
and feet. On which arteries at norm there must be the greatest systolic pressure?
Humera
Radial
Aorta
Femoral
The dorsal artery of foot
It is known, that medium speed of pulse wave in vessels makes 4-10 m/s. What factors the speed of
pulse wave depends on?
Power of cardiac contractions
Volume of circulatory blood
Size of stroke volume
Viscosity of blood
Elasticity of vessel walls
At palpatory examination of pulse, the decrease of filling of pulse was found. What factor this
property of pulse depends on mainly?
volume of circulatory blood
elasticity of arterial walls
speed of blood-flow
peripheral resistance of vessels
From heart beat rate
Doctors have found the arterial hypertension, with the level of pressure - 150/90 mm Hg, in a 40
years old patient. What factors can affect the increase of pressure?
Volume of circulatory blood
Peripheral resistance of vessels
Viscosity of blood
D.
E. *
93.
A. *
B.
C.
D.
E.
94.
A. *
B.
C.
D.
E.
95.
A.
B.
C. *
D.
E.
96.
A.
B.
C.
D. *
E.
97.
A.
B. *
C.
D.
E.
98.
A.
B.
C. *
D.
E.
99.
A.
B. *
C.
Elasticity of vessel walls
All of the factors
What is the main force of the blood-flow in vessels?
Gradient of pressure at the beginning and at the end of vessels
Gradient of resistance at the beginning and at the end of vessels
Difference of pressure and resistance in vessels
Gradient of volume at the beginning and at the end of vessels
Difference of the diameter at the beginning and at the end of vessels
Doctors had a necessity to estimate the elasticity of large arterial vessels. Which one of the methods
of research should be used for this purpose?
Sphygmography
Electrocardiography
Phonocardiography
Polycardiography
Vessel scanning
Functional types of vessels are:
Elastic, arterial, exchange, capacitable, venous
Capillary, resistive, capacitable, exchange, venous, muscular
Amortizing, resistive, exchange, shunting, capacitable
Muscular, capacitable, pulsating, shunting, amortizing
Arterial, venous, capillary, resistive, shunting
Doctors have conducted the registration of a sphygmogram of a patient with insufficiency of aortic
valves. How can a sphygmoraphic curve change in this case?
amplitude of anacrote will decrease
duration of catacrote will be increased
amplitude of the dicrotic rise will be increased
amplitude of the dicrotic rise will be decreased
duration of anacrote and catacrote will decrease
At palpatory research of pulse, the increase of tension of pulse is set. What factor this property of
pulse depends on mainly?
volume of circulatory blood
elasticity of arterial walls
speed of blood-flow
peripheral resistance of vessels
From heart beat rate
What factor the speed of blood flow in vessels depends from mainly?
length of vessel
size of stroke volume
total area of the transversal section of vessel
of stroke volume
elasticity of vessel walls
Microcirculation has such functional groups of vessels:
Capillaries, resistive postcapillaries, arteriols, precapillaries, small arteries
Arterioles, precapillaries, capillaries, vessels-shunts, resistive postcapillaries, venules, lymphatic
vessels
Capillaries, venules, arterioles, precapillaries, postcapillaries, small veins and arteries
D.
E.
100.
A.
B.
C.
D.
E. *
101.
A. *
B.
C.
D.
E.
102.
A. *
B.
C.
D.
E.
103.
A.
B.
C. *
D.
E.
104.
A.
B.
C.
D. *
E.
105.
A.
B.
C. *
D.
E.
Capillaries, venules, precapillaries, postcapillaries, small veins and arteries
Arteriols, precapillaries, capillaries, vessels-shunts, resistive postcapillaries, venules
Name 5 types of vessels, which belong to the microcirculation.
Capillaries, arteriols, precapillaries, amortizing, vessels-shunts
Resistive postcapillaries, capacitable, capillaries, arteriols, precapillaries
Vessels-shunts, precapillaries, amortizing, arteriols, postcapillaries
Capacitable vessels, amortizing, arteriols, capillaries, venules
Arteriols, precapillaries, capillaries, vessels-shunts, resistive postcapillaries
What substances pass through the wall of capillaries of spleen?
Plasma and unharmed formed elements
Plasma and fragments of blood cells
Plasma without high molecular albumens
Plasma without fibrinogen
Plasma without albumens
During the ultrastructure examination of a tissue sample, researchers found that the walls of
capillaries consist of a layer of endothelial cells, with small spaces ( less than 0.1mcm ) between
them. What type these capillaries belong to?
Visceral
Plasmatic
Somatic
Sinusoid
Lymphatic
A patient B., 68 years old, has ischemic heart disease with insufficiency of blood-circulation of ІІІ
grade. At the examination, the edemata is set on the lower extremities. What changes of
microcirculation could lead to development of edemata?
Increase of oncotic pressure of blood
Decrease of oncotic pressure of blood
Increase of hydrostatical pressure in the venous end of capillary
Decrease of hydrostatical pressure in the venous end of capillary
Decrease of hydrostatical pressure in the arterial end of capillary
While conducting the test of Conchalovsky the increase of quantity of point hemorrhages is exposed
on a forearm in a circle with a diameter of 5 cm. What is the normal quantity of hemorrhages in this
circle?
Up to 5
Up to 3
Up to 8
Up to 10
Up to 12
While conducting the electronic microscopy of a sample of tissue, capillaries that have the
continuous layer of endothelial cells and a basal membrane were found. Sample of what tissue was
examined?
Kidneys
Liver
Cerebrum
Bone marrow
Intestine
106.
A.
B. *
C.
D.
E.
107.
A.
B. *
C.
D.
E.
108.
A.
B.
C. *
D.
E.
109.
A.
B. *
C.
D.
E.
110.
A. *
B.
C.
D.
E.
111.
A.
B.
C. *
D.
E.
While conducting the electronic microscopy of a sample of tissue, capillaries that have the fenesters
in endothelium (more than 0,1 mcm wide) with a practically absent basal membrane were found.
Sample of what tissue was examined?
Cerebrum
Bone marrow
Muscle
Skin
Urinary bladder
During the experiment, by a direct method, it was established that the size of hydrostatical pressure in
the arterial end of capillary was evened 36 mm Hg. How will filtration and reabsorbtion change in a
microcirculation here?
Filtration will be increased
Filtration will decreased
Reabsorbtion will decreased
Filtration and reabsorbtion will be increased
Filtration and reabsorbtion will decrease
During the experiment, it was set by a direct method, that the size of hydrostatical pressure in the
venous end of capillary was evened 21 mm Hg. How will filtration and reabsorbtion change in a
microcirculation here?
reabsorbtion will be increased, filtration will decrease
Filtration will decrease
reabsorbtion will decrease
Filtration and reabsorbtion will be increased
Filtration and reabsorbtion will decrease
The capillaroscopia discovered a plenty of capillaries, that do not contain red blood cells. How are
such capillaries called?
Perfused
Plasmatic
Transitional
Functioning
Unfunctioning
Give determination to the phenomenon of Foreus-Lindkvist:
With the decrease of the diameter of vessel the speed of blood-flow grows
With the increase of the diameter of vessel the speed of blood-flow grows
With the decrease of the diameter of vessel the speed of blood-flow diminishes
With the decrease of the diameter, peripheral resistance of vessels grows
With the decrease of the diameter peripheral resistance is increasing
During the ultrastructure examination of a tissue sample, researchers found that the walls of
capillaries consist of a continuous layer of endothelial cells and basal membrane. What type these
capillaries belong to?
Visceral
Plasmatic
Somatic
Sinusoid
Lymphatic
112.
A.
B.
C.
D. *
E.
113.
A.
B. *
C.
D.
E.
114.
A.
B.
C. *
D.
E.
115.
A.
B. *
C.
D.
E.
116.
A.
B. *
C.
D.
E.
117.
A.
B.
C.
D. *
E.
During the ultrastructure examination of a tissue sample, researchers found that the walls of
capillaries consist of a layer of endothelial cells, with spaces ( more than 0.1mcm ) between them and
practically absent basal membrane. What type these capillaries belong to?
Visceral
Plasmatic
Somatic
Sinusoid
Lymphatic
A 42 years old patient C. with chronic glomerulonephritis, considerable proteinuria is set (excretion
of albumen with urine). At the examination, doctors found the edemata of face. What changes in a
microcirculation could lead to development of edemata?
Increase of oncotic pressure of blood
Decrease of oncotic pressure of blood
Increase of hydrostatical pressure in the venous end of capillary
Decrease of hydrostatical pressure in the venous end of capillary
Decrease of hydrostatical pressure in the arterial end of capillary
While conducting the electronic microscopy of a sample of tissue, capillaries that have the
continuous layer of endothelial cells and a basal membrane were found. Sample of what tissue was
examined?
Kidneys
Liver
Muscle
Bone marrow
Intestine
While conducting the electronic microscopy of a sample of tissue, capillaries that have the fenesters
in endothelium and pores in a basal membrane, were found. Sample of what tissue was examined?
Cerebrum
Liver
Muscle
Skin
Urinary bladder
During the experiment it is set by a direct method, that the size of hydrostatical pressure in the
arterial end of capillary was evened about 26 mm Hg. How will filtration and reabsorbtion change in
a microcirculation here?
Filtration will be increased
Filtration will decreased
Reabsorbtion will decreased
Filtration and reabsorbtion will be increased
Filtration and reabsorbtion will decrease
As a result of the protracted starvation, at a 36 years old man, the quantity of albumens decreased in
plasma of blood to 25 g/l. It led to the decline of oncotic pressure of plasma of blood to 20 mm Hg.
How will filtration and reabsorbtion change in a microcirculation here?
Filtration will decrease, reabsorbtion will be increased
Filtration will decrease
reabsorbtion will decrease
Filtration will be increased, reabsorbtion will diminish
Filtration and reabsorbtion will decrease
118.
A.
B. *
C.
D.
E.
119.
A.
B. *
C.
D.
E.
120.
A.
B.
C.
D. *
E.
121.
A.
B.
C.
D. *
E.
122.
A.
B.
C. *
D.
E.
123.
A.
B. *
C.
D.
E.
During the experiment, it was set by a direct method, that the size of hydrostatical pressure in the
venous end of capillary was evened 15 mm Hg. How will filtration and reabsorbtion change in a
microcirculation here?
Filtration will decrease
Reabsorbtion will increase
Filtration and reabsorbtion will increase
Filtration and reabsorbtion will decrease
Reabsorbtion will decrease, filtration will be increased
Name 5 factors which cause the movement of blood in veins
Difference of pressure in the venous system, quantity of circulatory blood, tone of vessels, viscosity
of blood, resistance in vessels
Difference of pressure in the venous system, negative pressure in a thoracic cavity, contractions of
skeletal muscles, diaphragmal pump, contraction of walls of veins
Difference of pressure, valves of veins, contraction of muscles
Negative pressure in a thorax, diaphragmal pump, difference of pressure, valves of veins, contraction
of muscles
Difference of pressure, valves of veins, tonus of veins, contraction of muscles
During conducting of march test, the expansion and increase of blood-filling of superficial veins of
right shin (leg) was observed. What is the most credible reason of this?
Insufficiency of valves of superficial veins
Insufficiency of valves of perforantive veins
Violation of permeability of superficial veins
Violation of permeability of deep veins
Violation of permeability of perforantive veins
At the analysis of phlebogram, the decline of amplitude of wave "a" was observed. What process
forms this wave on the phlebogram?
Relaxation of ventricles
Transmission of pulsation of carotid on a jugular vein
Systole of the right ventricle
Systole of the right atrium
Systole of the left atrium
While examining a 42 years old patient C., doctors have suspected the violation of permeability of
deep veins of right leg. What test should be conducted in order to prove this diagnosis?
Test Troyanova-Trendelenbourga
Test Sheynisa
Test of Delbe -Pertesa
Test Valsalvi
Test with the single physical loading.
While examining a 33 years old patient B., doctors have suspected a violation of function of valves of
perforantive veins. What test should be conducted in order to prove this diagnosis?
Test Troyanova-Trendelenbourga
Test Sheynisa
Test of Delbe -Pertesa
Test Valsalvi
Test with the single physical loading
124.
A. *
B.
C.
D.
E.
125.
A.
B. *
C.
D.
E.
126.
A.
B.
C.
D. *
E.
127.
A.
B.
C.
D.
E. *
128.
A.
B.
C.
D.
E. *
129.
A.
B. *
C.
D.
E.
130.
A.
B.
To the patient, for estimation of the permeability and function of valves of veins, it was
recommended to do tourniquet tests. During the test of Troyanov-Trendelenbourg, the expansion of
superficial veins was observed. For estimation of valves of what veins this test is usually used?
Superficial
Deep
Caval
Cranial
Perforantive
While conducting the test of Sheynise, expansion and increase of blood-filling of superficial veins of
right leg was observed. What is the most possible reason of this state?
Insufficiency of valves of superficial veins
Insufficiency of valves of perforantive veins
Violation of permeability of superficial veins
Violation of permeability of deep veins
Violation of permeability of perforantive veins
What type of transport provides the penetration of high molecular albumens in a lymph?
Diffusion
Osmosis
Primary active transport
Pinocytosis
Filtration
In a laboratory, the time of lymph-clotting was determined and was set, that it is higher normal one.
What is the normal duration of clotting of lymph?
Up to 3 min
Up to 5 min
5 - 10 min
60-120 sec
10-15 min
Name 5 functions of lymph
Protective, thermoregulatory, excretion, transport, filtration
Barrier, excretory, protective, homeostatic, transference of nutritives
Drainage, support of volume of tissue liquid
Excretion, protection, drainage, homeostatic, barrier
Support of volume of tissue liquid, transference of nutritives, filtration, barrier, participation in the
immunological reactions
The 68 years old patient Z., with ischemic illness of the heart, had his venous pressure measured,
which in vena ulnaris made 340 mm of waters column. What size of pressure must is normal in this
case?
45-85 mm of mercury column
45-120 mm of water column.
45-120 mm of mercury column
300-400 mm of mercury column
Up to 139 mm of mercury column.
At a 63 years old patient B., with arterial hypertension, the increase of amplitude of wave "c" was set
on the phlebogram. What process forms this wave on the phlebogram?
Transmission of pulsation of carotid on a jugular vein
Systole of the right ventricle
C. *
D.
E.
131.
A. *
B.
C.
D.
E.
132.
A.
B.
C.
D. *
E.
133.
A.
B. *
C.
D.
E.
134.
A. *
B.
C.
D.
E.
135.
A.
B.
C.
D.
E. *
136.
A.
B.
C. *
Systole of the right atrium
Systole of the left atrium
Relaxation of ventricles.
33 years old patient, has complains on pains and swelling of the left shin (leg). She was suspected to
have the violation of functions of valves of v. saphena magna. What test should be conducted in order
to prove this diagnosis?
Test Troyanova-Trendelenbourga
Test Sheynisa
Test of Delbe -Pertesa
Test Valsalvi
Test with the single physical loading
To the patient, for estimation of the permeability and function of valves of veins, it was
recommended to do tourniquet tests. During the three-tourniquet test, the expansion of superficial
veins was observed. For estimation of valves of what veins this test is usually used?
Superficial
Deep
Caval
Perforantive
Cranial
To the patient, for estimation of the permeability and function of valves of veins, it was
recommended to do tourniquet tests. During the march test, the expansion of superficial veins was
observed. For estimation of valves of what veins this test is usually used?
Superficial
Deep
Caval.
Cranial
Perforantive
While conducting the test of Troyanov-Trendelenbourg, the expansion and increase of blood-filling
of superficial veins of right shin was observed. What is the most possible reason of this state?
Insufficiency of valves of superficial veins
Insufficiency of valves of perforantive veins
Violation of permeability of superficial veins
Violation of passing ability of deep veins
Violation of permeability of perforantive veins
In a laboratory, the research of definite liquid was conducted. Cations, anions, fibrinogen (1,1 g/l),
leucocytes (lymphocytes - 90 %, neutrophills - 10 %). Research of what liquid was conducted?
Pleural liquid
Spinal liquid
Peripheral lymph
Blood
Central lymph
It is known, that during the hard physical work, the formation of lymph is considerably increased.
What is the reason of such increase?
Increase of oncotic pressure of plasma of blood
Reduction of oncotic pressure of plasma of blood
Increase of osmotic pressure of tissue liquid
D.
E.
137.
A.
B.
C. *
D.
E.
138.
A.
B.
C.
D. *
E.
139.
A. *
B.
C.
D.
E.
140.
A.
B. *
C.
D.
E.
141.
A.
B.
C.
D. *
E.
142.
A. *
B.
C.
D.
E.
143.
Reduction of osmotic pressure of tissue liquid
Increase of osmotic pressure of plasma of blood
What quantity of lymph is normally produced in an organism daily?
Near 20 ml
Near 200 ml
Near 2000 ml
Near 20 000 ml
Near 5 000 ml
On the lesson, students have conducted a test with the single overloading. After overloading, they
measured the arterial pressure and found that it’s level exceeded the level which has been estimated
before the test. However, the level of pulse pressure (PP) did not change. What formula should be
used to calculate PP?
APsyst - systolic arterial pressure; APdiast - diastolic arterial pressure; PP - pulse pressure.
PP = APsyst. · APdiast
PP = APsyst.: APdiast
PP = APsyst. + APdiast
PP = APsyst. - APdiast
PP = (APsyst. - APdiast) : 3
On the experiment on a dog, it was set that a law of Starling for a heart represents:
Ability of heart to increase the power of contractions during the increase of filling of his chambers
with blood.
Correlation of consumption of oxygen by myocardium with work, that is being done
Correlation of volume of right atrium with the frequency of cardiac contractions
Correlation of the cardiac ejection with the peripheral resistance
Change of tonus of veins and skeletal muscles
How does the work of heart of patients with the diabetes mellitus changes?
Heart beat rate will be increased
Heart beat rate will decrease
Power of cardiac contractions will be increased
Speed of conducting of excitation will decrease
Speed of conducting of excitation will be increased
On the lesson, through the isolated heart of frog the 3 % solution of CaCl2 was passed by. How will
the heart work change?
power of cardiac contractions will decrease, heart beat rate will increase
speed of conducting of excitation will increase
heart will stop in a diastole
heart will stop in a systole
decrease of excitability
The Boudichi phenomenon sets the dependence of power of cardiac contractions on:
Heart beat rate
degree of dilatation of cardiomyocites
pressure in an aorta
volume of ventricles
speed of conducting of excitation
Doctors have prescribed the β-adrenostimulators to a patient N. with bronchial asthma. How will the
work of heart after the reception of these medications change?
A.
B.
C. *
D.
E.
144.
A. *
B.
C.
D.
E.
145.
A. *
B.
C.
D.
E.
146.
A. *
B.
C.
D.
E.
147.
A. *
B.
C.
D.
E.
148.
A. *
B.
C.
D.
E.
149.
A.
B. *
C.
D.
duration of the diastole will be increased
duration of the systole will be increased
duration of the diastole will decrease
duration of the systole will decrease
delay time of conducting of excitation in atrio-ventricular node will be increased
At a patient with a syndrome of Itsenco-Coushing the production of glucocorticoids was
considerably increased. How will the work of heart change?
heart beat rate will increase
heart beat rate will decrease
speed of conducting of excitation in atrio-ventricular node will decrease
power of cardiac contractions will decrease
the work of heart will not change
On the lesson, through the isolated heart of frog the solution was passed by. It caused the increase of
heart beat rate. What solution was used?
Adrenalin
1 % solution of NaCl
Potassium chloride
Acetylcholine
NaHCO3
On the lesson, students placed the isolated heart of frog in solution. No changes in heart work
happened. In what solution was the heart placed?
1 % solution of NaCl.
3 % solution of HCl
1 % solution of CaCl2
3 % solution of KCl
1 % solution of NaHCO3
Patients with heart pathologies often have the increased blood level of a hormone which decreases
the reabsorbsion of Na+ and water in tubules, and is produced in a heart. What hormone has such
action?
Atrial sodium uretic hormone
Rennin
Aldosteron
Vasopressin
Adrenalin
Doctors during the inspection defined the presence of tachycardia in the state of rest. What can be its
reason?
Increased function of thyroid gland
Increased function of cortex of adrenal glands
Increased permeability of Cl-canals of myocardium
Increased tonus of the parasympathic system
Change of all listed factors
At a patient B., with the increased function of thyroid gland, the increase of heart beat rate is set.
What is the mechanism of the thyroid hormone influence on the heart?
decrease of quantity of B-adrenoreceptors
increase of quantity of B-adrenoreceptors
increase of quantity of N-cholinoreceptors
decrease of quantity of alfa-adrenoreceptors
E.
150.
A.
B. *
C.
D.
E.
151.
A.
B.
C. *
D.
E.
152.
A.
B.
C.
D. *
E.
153.
A.
B.
C.
D. *
E.
154.
A. *
B.
C.
D.
E.
155.
A. *
B.
C.
D.
E.
156.
A. *
increase of quantity of M- cholinoreceptors
A 56 years old patient K, with arterial hypertension, has used diuretics for a long time, that resulted
in violation of cardiac rhythm. By the change of concentration of what ion is this state conditioned?
increase of K+
decrease of K+
increase of Na+
decrease of Na+
decrease of Ca2+
In what conditions does the atrial natriuretic peptide excretion increases (by the incretory auricle
atrium cells)?
with the increase of concentration of K+
with the decrease of concentration of K+
with the increase of volume of circulatory blood
with the decrease of the volume of circulatory blood
with the increase of level of Ca2+.
On the lesson, the students placed the isolated heart of frog in a solution. The heart stop in a diastole
happened. What solution was heart placed in?
1 % solution of NaCl.
3 % solution of NaCl
1 % solution of CaCl2
3 % solution of KCl
1 % solution of NaHCO3
On the lesson, through the isolated heart of frog the solution was passed by. It caused the decrease of
heart beat rate. What solution was used?
Adrenalin
Histamine
Calcium chloride
Acetylcholine
1 % NaHCO3.
At a patient with a syndrome of Cone the production of aldosteron was considerably increased. How
will the work of heart change?
heart beat rate will increase
heart beat rate will decrease
speed of conducting of excitation in atrio-ventricular node will decrease
power of cardiac contractions will decrease
the work of heart will not change
It is known, that the heart beat rate decreases under the action of acetylcholine. What mechanism
predetermines it?
duration of phase of spontaneous diastolic depolarization is increased
duration of phase of spontaneous diastolic depolarization decreases
speed of conducting of excitation to atrio-ventricular node is increased
duration of action potential is increased
duration of action potential is increased
Doctors have prescribed the β-adrenoblockators to a patient K. with arterial hypertension. How will
the work of heart after the reception of these medications change?
duration of the diastole will be increased
B.
C.
D.
E.
157.
A. *
B.
C.
D.
E.
158.
A. *
B.
C.
D.
E.
159.
A.
B.
C.
D.
E. *
160.
A.
B.
C. *
D.
E.
161.
A.
B. *
C.
D.
E.
162.
A.
B.
C.
D.
E. *
duration of the systole will be increased
duration of the diastole will decrease
duration of the systole will decrease
delay time of conducting of excitation in atrio-ventricular node will be decreased
During the experiment on an animal, the right vagus nerve was cut. What changes would be
observed?
Increase of heart beat rate
Increase of power of cardiac contractions
Increase of speed of conducting of the excitation in an atrio-ventricular node
Decrease of heart beat rate
Decrease of power of cardiac contractions
On the lesson, students inflicted the strong irritation of vagus nerve of a rat, that caused the heart to
stop. What is the nature of this phenomenon?
activation of K+ - channels of the cells of sinoatrial node
activation of Na+ - channels of the cells of sinoatrial node
inactivation of the Na+ - channels of the cells of sinoatrial node
activation of the Ca2+ - channels of the cells of sinoatrial node
inactivation of the Ca2+ - channels of the cells of sinoatrial node
It is known, that in emergency therapy for the removal of the attack of paroxysmal tachycardia,
doctors use the reflex, that is inducted by pressing on the eyeballs. How is this reflex called?
Hering’s
Holt’s
Bekhterev’s
Helmholt’s
Ashner’s
On practical the students during the experiment conducted electro-stimulation of the left vagus nerve.
What effect here will be observed?
Positive inotropic effect
Positive chronotropic effect
Negative inotropic effect
Positive dromotropic effect
No bathmotropic effect
On the lesson, students inducted the reflex by inflicting the strikes on the anterior abdominal wall of
frog. How is this reflex called?
Hering’s
Holt’s
Bekhterev’s
Valsalve’s
Ashner’s
It is known, that during the hard physical work, the heart beat rate is increasing. What receptors
conduct the afferent information in this case?
Mechanoreceptors of vessels
Chemoreceptors of muscles
Chemoreceptors of carotid sinus
Mechanoreceptors of the right atrium
Proprioreceptors
163.
A.
B.
C.
D. *
E.
164.
A.
B. *
C.
D.
E.
165.
A.
B.
C. *
D.
E.
166.
A. *
B.
C.
D.
E.
167.
A. *
B.
C.
D.
E.
168.
A.
B.
C.
D.
E. *
169.
A.
B.
During the acute experiment on an animal, the superior thoracic segments of spinal cord were
destroyed. How will the heart activity change in such conditions?
Increase of heart beat rate
Increase of power of cardiac contractions
Increase of conducting speed of excitation in atrio-ventricular node
Decrease of heart beat rate
Decrease of end-diastolic volume
On the lesson, students observed the respiratory arrhythmia. Thus on the monitor of cardioscope was
evidently, that at inhalation the heart beat rate was increasing, and at exhalation - decreasing. How
will the stroke volume (SV) change at the inhalation and exhalation?
At inhalation will increase, at exhalation - will decrease
At inhalation will decrease, at exhalation - will increase
Will increase at inhalation, at exhalation will not change
Will increase at exhalation, at inhalation will not change
Changes of stroke volume at inhalation and exhalation would not be observed
It is known, that the frequency of cardiac contractions decreases during a sleep. What mechanism
causes such changes?
Inhibition of sensory areas of the cerebral cortex
Inhibition of motor areas of the cerebral cortex
Increase of tonus of n.vagus
Increase of tonus of the sympathetic nervous system
Inhibition of neurons of middle hypothalamus
The dicrotic wave on sphygmogram appears as a result of:
Push of blood into closed semilunar valves of aorta
Push of blood into closed semilunar valves of pulmonary trunk
Transition of blood from ventricles in an aorta
Push of blood into closed mitral valves
Push of blood into closed tricuspid valves
One of important haemodynamic indexes is the linear speed of blood-flow. What factor does this
index depend on mainly?
Total area of cross-section of vessels
Length of vessel
Viscosity of blood
Size of stroke volume
Gradient of pressure
During the prophylactic examination of a 25 years old man, arterial pressure was measured on hands
and feet. On which arteries at norm there must be the greatest systolic pressure?
Humera
Radial
Aorta
Femoral
The dorsal artery of foot
It is known, that medium speed of pulse wave in vessels makes 4-10 m/s. What factors the speed of
pulse wave depends on?
Power of cardiac contractions
Volume of circulatory blood
C.
D.
E. *
170.
A. *
B.
C.
D.
E.
171.
A.
B.
C.
D.
E. *
172.
A. *
B.
C.
D.
E.
173.
A. *
B.
C.
D.
E.
174.
A. *
B.
C.
D.
E.
175.
A.
B.
C. *
D.
E.
176.
A.
Size of stroke volume
Viscosity of blood
Elasticity of vessel walls
At palpatory examination of pulse, the decrease of filling of pulse was found. What factor this
property of pulse depends on mainly?
volume of circulatory blood
elasticity of arterial walls
speed of blood-flow
peripheral resistance of vessels
From heart beat rate
Doctors have found the arterial hypertension, with the level of pressure - 150/90 mm Hg, in a 40
years old patient. What factors can affect the increase of pressure?
Volume of circulatory blood
Peripheral resistance of vessels
Viscosity of blood
Elasticity of vessel walls
All of the factors
What is the main force of the blood-flow in vessels?
Gradient of pressure at the beginning and at the end of vessels
Gradient of resistance at the beginning and at the end of vessels
Difference of pressure and resistance in vessels
Gradient of volume at the beginning and at the end of vessels
Difference of the diameter at the beginning and at the end of vessels
During the examination, the rise of diastolic pressure was found in a 45 years old patient. Exceeding
of what level of pressure testifies it?
89 mm Hg
90 mm Hg
95 mm Hg
70 mm Hg
96 mm Hg
Doctors had a necessity to estimate the elasticity of large arterial vessels. Which one of the methods
of research should be used for this purpose?
Sphygmography
Electrocardiography
Phonocardiography
Polycardiography
Vessel scanning
Functional types of vessels are:
Elastic, arterial, exchange, capacitable, venous
Capillary, resistive, capacitable, exchange, venous, muscular
Amortizing, resistive, exchange, shunting, capacitable
Muscular, capacitable, pulsating, shunting, amortizing
Arterial, venous, capillary, resistive, shunting
Doctors have conducted the registration of a sphygmogram of a patient with insufficiency of aortic
valves. How can a sphygmoraphic curve change in this case?
amplitude of anacrote will decrease
B.
C.
D.
duration of catacrote will be increased
amplitude of the dicrotic rise will be increased
amplitude of the dicrotic rise will be decreased
duration of anacrote and catacrote will decrease
What factor the speed of blood flow in vessels depends from mainly?
length of vessel
size of stroke volume
total area of the transversal section of vessel
of stroke volume
elasticity of vessel walls
At palpatory research of pulse, the increase of tension of pulse is set. What factor this property of
pulse depends on mainly?
volume of circulatory blood
elasticity of arterial walls
speed of blood-flow
peripheral resistance of vessels
From heart beat rate
During the examination, the rise of systolic pressure was found in a 45 years old patient. Exceeding
of what level of pressure testifies it?
129 mm Hg
139 mm Hg
149 mm Hg
159 mm Hg
130 mm Hg
Name 5 types of vessels, which belong to the microcirculation.
Capillaries, arteriols, precapillaries, amortizing, vessels-shunts
Resistive postcapillaries, capacitable, capillaries, arteriols, precapillaries
Vessels-shunts, precapillaries, amortizing, arteriols, postcapillaries
Capacitable vessels, amortizing, arteriols, capillaries, venules
Arteriols, precapillaries, capillaries, vessels-shunts, resistive postcapillaries
What substances pass through the wall of capillaries of spleen?
Plasma and unharmed formed elements
Plasma and fragments of blood cells
Plasma without high molecular albumens
Plasma without fibrinogen
Plasma without albumens
During the ultrastructure examination of a tissue sample, researchers found that the walls of
capillaries consist of a layer of endothelial cells, with small spaces ( less than 0.1mcm ) between
them. What type these capillaries belong to?
Visceral
Plasmatic
Somatic
Sinusoid
E.
Lymphatic
C.
D. *
E.
177.
A.
B.
C. *
D.
E.
178.
A.
B. *
C.
D.
E.
179.
A.
B. *
C.
D.
E.
180.
A.
B.
C.
D.
E. *
181.
A. *
B.
C.
D.
E.
182.
A. *
B.
183.
A.
B.
C. *
D.
E.
184.
A.
B.
C.
D. *
E.
185.
A.
B.
C. *
D.
E.
186.
A.
B. *
C.
D.
E.
187.
A.
B. *
C.
D.
E.
188.
A.
B.
C. *
D.
E.
189.
A patient B., 68 years old, has ischemic heart disease with insufficiency of blood-circulation of ІІІ
grade. At the examination, the edemata is set on the lower extremities. What changes of
microcirculation could lead to development of edemata?
Increase of oncotic pressure of blood
Decrease of oncotic pressure of blood
Increase of hydrostatical pressure in the venous end of capillary
Decrease of hydrostatical pressure in the venous end of capillary
Decrease of hydrostatical pressure in the arterial end of capillary
While conducting the test of Conchalovsky the increase of quantity of point hemorrhages is exposed
on a forearm in a circle with a diameter of 5 cm. What is the normal quantity of hemorrhages in this
circle?
Up to 5
Up to 3
Up to 8
Up to 10
Up to 12
While conducting the electronic microscopy of a sample of tissue, capillaries that have the
continuous layer of endothelial cells and a basal membrane were found. Sample of what tissue was
examined?
Kidneys
Liver
Cerebrum
Bone marrow
Intestine
While conducting the electronic microscopy of a sample of tissue, capillaries that have the fenesters
in endothelium (more than 0,1 mcm wide) with a practically absent basal membrane were found.
Sample of what tissue was examined?
Cerebrum
Bone marrow
Muscle
Skin
Urinary bladder
During the experiment, by a direct method, it was established that the size of hydrostatical pressure in
the arterial end of capillary was evened 36 mm Hg. How will filtration and reabsorbtion change in a
microcirculation here?
Filtration will be increased
Filtration will decreased
Reabsorbtion will decreased
Filtration and reabsorbtion will be increased
Filtration and reabsorbtion will decrease
During the experiment, it was set by a direct method, that the size of hydrostatical pressure in the
venous end of capillary was evened 21 mm Hg. How will filtration and reabsorbtion change in a
microcirculation here?
reabsorbtion will be increased, filtration will decrease
Filtration will decrease
reabsorbtion will decrease
Filtration and reabsorbtion will be increased
Filtration and reabsorbtion will decrease
Give determination to the phenomenon of Foreus-Lindkvist:
A. *
B.
C.
D.
E.
190.
A.
B.
C. *
D.
E.
191.
A.
B.
C.
D. *
E.
192.
A.
B. *
C.
D.
E.
193.
A.
B.
C. *
D.
E.
194.
A.
B. *
C.
D.
E.
195.
A.
B. *
With the decrease of the diameter of vessel the speed of blood-flow grows
With the increase of the diameter of vessel the speed of blood-flow grows
With the decrease of the diameter of vessel the speed of blood-flow diminishes
With the decrease of the diameter, peripheral resistance of vessels grows
With the decrease of the diameter peripheral resistance is increasing
During the ultrastructure examination of a tissue sample, researchers found that the walls of
capillaries consist of a continuous layer of endothelial cells and basal membrane. What type these
capillaries belong to?
Visceral
Plasmatic
Somatic
Sinusoid
Lymphatic
During the ultrastructure examination of a tissue sample, researchers found that the walls of
capillaries consist of a layer of endothelial cells, with spaces ( more than 0.1mcm ) between them and
practically absent basal membrane. What type these capillaries belong to?
Visceral
Plasmatic
Somatic
Sinusoid
Lymphatic
A 42 years old patient C. with chronic glomerulonephritis, considerable proteinuria is set (excretion
of albumen with urine). At the examination, doctors found the edemata of face. What changes in a
microcirculation could lead to development of edemata?
Increase of oncotic pressure of blood
Decrease of oncotic pressure of blood
Increase of hydrostatical pressure in the venous end of capillary
Decrease of hydrostatical pressure in the venous end of capillary
Decrease of hydrostatical pressure in the arterial end of capillary
While conducting the electronic microscopy of a sample of tissue, capillaries that have the
continuous layer of endothelial cells and a basal membrane were found. Sample of what tissue was
examined?
Kidneys
Liver
Muscle
Bone marrow
Intestine
While conducting the electronic microscopy of a sample of tissue, capillaries that have the fenesters
in endothelium and pores in a basal membrane, were found. Sample of what tissue was examined?
Cerebrum
Liver
Muscle
Skin
Urinary bladder
During the experiment it is set by a direct method, that the size of hydrostatical pressure in the
arterial end of capillary was evened about 26 mm Hg. How will filtration and reabsorbtion change in
a microcirculation here?
Filtration will be increased
Filtration will decreased
C.
D.
E.
196.
A.
B.
C.
D. *
E.
197.
A.
B. *
C.
D.
E.
198.
A.
B. *
C.
D.
E.
199.
A.
B.
C.
D. *
E.
200.
A.
B.
C. *
D.
E.
201.
A.
B. *
C.
Reabsorbtion will decreased
Filtration and reabsorbtion will be increased
Filtration and reabsorbtion will decrease
As a result of the protracted starvation, at a 36 years old man, the quantity of albumens decreased in
plasma of blood to 25 g/l. It led to the decline of oncotic pressure of plasma of blood to 20 mm Hg.
How will filtration and reabsorbtion change in a microcirculation here?
Filtration will decrease, reabsorbtion will be increased
Filtration will decrease
reabsorbtion will decrease
Filtration will be increased, reabsorbtion will diminish
Filtration and reabsorbtion will decrease
During the experiment, it was set by a direct method, that the size of hydrostatical pressure in the
venous end of capillary was evened 15 mm Hg. How will filtration and reabsorbtion change in a
microcirculation here?
Filtration will decrease
Reabsorbtion will increase
Filtration and reabsorbtion will increase
Filtration and reabsorbtion will decrease
Reabsorbtion will decrease, filtration will be increased
Name 5 factors which cause the movement of blood in veins
Difference of pressure in the venous system, quantity of circulatory blood, tone of vessels, viscosity
of blood, resistance in vessels
Difference of pressure in the venous system, negative pressure in a thoracic cavity, contractions of
skeletal muscles, diaphragmal pump, contraction of walls of veins
Negative pressure in a thorax, diaphragmal pump, difference of pressure, valves of veins, contraction
of muscles
Difference of pressure, valves of veins, contraction of muscles
Difference of pressure, valves of veins, tonus of veins, contraction of muscles
During conducting of march test, the expansion and increase of blood-filling of superficial veins of
right shin (leg) was observed. What is the most credible reason of this?
Insufficiency of valves of superficial veins
Insufficiency of valves of perforantive veins
Violation of permeability of superficial veins
Violation of permeability of deep veins
Violation of permeability of perforantive veins
While examining a 42 years old patient C., doctors have suspected the violation of permeability of
deep veins of right leg. What test should be conducted in order to prove this diagnosis?
Test Troyanova-Trendelenbourga
Test Sheynisa
Test of Delbe -Pertesa
Test Valsalvi
Test with the single physical loading.
While examining a 33 years old patient B., doctors have suspected a violation of function of valves of
perforantive veins. What test should be conducted in order to prove this diagnosis?
Test Troyanova-Trendelenbourga
Test Sheynisa
Test of Delbe -Pertesa
D.
Test Valsalvi
E.
202.
Test with the single physical loading
To the patient, for estimation of the permeability and function of valves of veins, it was
recommended to do tourniquet tests. During the test of Troyanov-Trendelenbourg, the expansion of
superficial veins was observed. For estimation of valves of what veins this test is usually used?
Superficial
Deep
Caval
Cranial
Perforantive
While conducting the test of Sheynise, expansion and increase of blood-filling of superficial veins of
right leg was observed. What is the most possible reason of this state?
Insufficiency of valves of superficial veins
Insufficiency of valves of perforantive veins
Violation of permeability of superficial veins
Violation of permeability of deep veins
Violation of permeability of perforantive veins
What type of transport provides the penetration of high molecular albumens in a lymph?
Diffusion
Osmosis
Primary active transport
Pinocytosis
Filtration
In a laboratory, the time of lymph-clotting was determined and was set, that it is higher normal one.
What is the normal duration of clotting of lymph?
Up to 3 min
Up to 5 min
5 - 10 min
60-120 sec
10-15 min
Name 5 functions of lymph
Protective, thermoregulatory, excretion, transport, filtration
Barrier, excretory, protective, homeostatic, transference of nutritives
Drainage, support of volume of tissue liquid
Excretion, protection, drainage, homeostatic, barrier
Support of volume of tissue liquid, transference of nutritives, filtration, barrier, participation in the
immunological reactions
The 68 years old patient Z., with ischemic illness of the heart, had his venous pressure measured,
which in vena ulnaris made 340 mm of waters column. What size of pressure must is normal in this
case? mm of mercury column
45-85
45-120 mm of water column.
45-120 mm of mercury column
300-400 mm of mercury column
Up to 139 mm of mercury column.
At a 63 years old patient B., with arterial hypertension, the increase of amplitude of wave "c" was set
on the phlebogram. What process forms this wave on the phlebogram?
A. *
B.
C.
D.
E.
203.
A.
B. *
C.
D.
E.
204.
A.
B.
C.
D. *
E.
205.
A.
B.
C.
D.
E. *
206.
A.
B.
C.
D.
E. *
207.
A.
B. *
C.
D.
E.
208.
A.
B.
C. *
D.
E.
209.
A. *
B.
C.
D.
E.
210.
A.
B.
C.
D. *
E.
211.
A.
B. *
C.
D.
E.
212.
A. *
B.
C.
D.
E.
213.
A.
B.
C.
D.
E. *
214.
A.
Transmission of pulsation of carotid on a jugular vein
Systole of the right ventricle
Systole of the right atrium
Systole of the left atrium
Relaxation of ventricles.
33 years old patient, has complains on pains and swelling of the left shin (leg). She was suspected to
have the violation of functions of valves of v. saphena magna. What test should be conducted in order
to prove this diagnosis?
Test Troyanova-Trendelenbourga
Test Sheynisa
Test of Delbe -Pertesa
Test Valsalvi
Test with the single physical loading
To the patient, for estimation of the permeability and function of valves of veins, it was
recommended to do tourniquet tests. During the tree-tourniquet test, the expansion of superficial
veins was observed. For estimation of valves of what veins this test is usually used?
Superficial
Deep
Caval
Perforantive
Cranial
To the patient, for estimation of the permeability and function of valves of veins, it was
recommended to do tourniquet tests. During the march test, the expansion of superficial veins was
observed. For estimation of valves of what veins this test is usually used?
Superficial
Deep
Caval.
Perforantive
Cranial
While conducting the test of Troyanov-Trendelenbourg, the expansion and increase of blood-filling
of superficial veins of right shin was observed. What is the most possible reason of this state?
Insufficiency of valves of superficial veins
Insufficiency of valves of perforantive veins
Violation of permeability of superficial veins
Violation of passing ability of deep veins
Violation of permeability of perforantive veins
In a laboratory, the research of definite liquid was conducted. Cations, anions, fibrinogen (1,1 g/l),
leucocytes (lymphocytes - 90 %, neutrophills - 10 %). Research of what liquid was conducted?
Pleural liquid
Spinal liquid
Peripheral lymph
Blood
Central lymph
It is known, that during the hard physical work, the formation of lymph is considerably increased.
What is the reason of such increase?
Increase of oncotic pressure of plasma of blood
B.
C. *
D.
E.
215.
A.
B.
C. *
D.
E.
216.
A.
B.
C.
D. *
E.
217.
A. *
B.
C.
D.
E.
218.
A. *
B.
C.
D.
E.
219.
A. *
B.
C.
D.
E.
220.
A. *
B.
C.
D.
221.
Reduction of oncotic pressure of plasma of blood
Increase of osmotic pressure of tissue liquid
Reduction of osmotic pressure of tissue liquid
Increase of osmotic pressure of plasma of blood
What quantity of lymph is normally produced in an organism daily?
Near 20 ml
Near 200 ml
Near 2000 ml
Near 20 000 ml
Near 5 000 ml
On the lesson, students have conducted a test with the single overloading. After overloading, they
measured the arterial pressure and found that it’s level exceeded the level which has been estimated
before the test. However, the level of pulse pressure (PP) did not change. What formula should be
used to calculate PP?
APsyst - systolic arterial pressure; APdiast - diastolic arterial pressure; PP - pulse pressure.
PP = APsyst. · APdiast
PP = APsyst.: APdiast
PP = APsyst. + APdiast
PP = APsyst. - APdiast
PP = (APsyst. - APdiast) : 3
?Mean circulatory pressure:
is the equilibrium pressure in vessels that exists with zero flow
is increased with decreases in blood volume
is increased with vasoconstriction
is increased with venodilation
is enhanced by increased cardiac contractility
Which of the following does NOT have a positive inotropic effect on cardiac muscle?
decrease external [Ca+2]
norepinephrine
decrease external [K+]
inhibition of the Na/K pump
increase frequency of stimulation
All of the following are true concerning the cutaneous circulation EXCEPT:
Increased environmental temperatures constrict blood vessels
Activation of sweat glands during heat exposure helps to dissipate heat.
Neural mechanisms are the most important control of cutaneous blood vessels.
The Triple Response of Lewis consists of a flare, resulting form an axon reflex to dilate neighboring
vessels.
AV anastomoses are found in hands, feet and some areas of the face..
Coronary blood flow is primarily regulated by:
metabolic demand
parasympathetic nervous system
epinephrine
sympathetic nervous system
During exercise, all of the following events occur EXCEPT:
A. *
B.
C.
D.
E.
222.
A. *
B.
C.
D.
E.
223.
A. *
B.
C.
D.
E.
224.
A. *
B.
C.
D.
E.
225.
A. *
B.
C.
D.
E.
226.
A. *
B.
C.
D.
E.
227.
A. *
B.
In normal individuals, the increase in cardiac output is attained primarily by increases in stroke
volume rather than heart rate
Skeletal muscle blood flow is increased during exercise due in part to vasodilator metabolite
production.
Starling's Law of the heart contributes to an increase in stroke volume only with very strenuous
exercise.
As mean arterial pressure increases during exercise, the increase in cardiac output must be greater
than the reduction in total peripheral resistance.
Total peripheral resistance decreases during exercise due to increased blood flow through skeletal
muscle.
Norepinephrine would have which of the following effects on the Sa node?
decrease rate of rise of phase 4 spontaneous depolarization increase Ca2+ conductance
make the resting membrane potential of the cell more negative (hyperpolarize)
increase K+ conductance
decrease rate of rise of phase 4 spontaneous depolarization increase
all of the above
The R wave of the ECG represents depolarization of:
ventricular miocardium
atrial muscle
base of the ventricles
SA node
interventricular septum
Which of the following sets of leads records the electrical activity of the heart in the transverse
plane?
precordial unipolar leads
precordial and augmented unipolar leads
standard bipolar leads
precordial unipolar and standard bipolar limb leads
augmented unipolar leads
Which of the following events is mediated by the activation of the Parasympathetic Nervous System?
contraction of sphincter muscles of the eye
relaxation of bronchioles
renin secretion from the kidney
constriction of arterioles
lipolysis
All the following statements about neurotransmitters of the Autonomic Nervous System are true
EXCEPT:
The neurotransmitter of sympathetic preganglionic nerve terminals is norepinephrine
Nerves that release acetylcholine are called cholinergic nerves
The chromaffin cells of the adrenal medulla releases mostly epinephrine and a small amount of
norepinephrine
Postganglionic sympathetic fibers to sweat glands release acetylcholine
All postganglionic parasympathetic fibers release acetylcholine
A TRUE statement about cholinergic receptors is:
atropine, an M receptor antagonist, blocks the effects of postganglionic sympathetic nerve
stimulation to sweat glands
blocking Nm receptors would prevent activation of the parasympathetic nervous system only
C.
D.
E.
228.
A. *
B.
C.
D.
E.
229.
A. *
B.
C.
D.
E.
230.
A. *
B.
C.
D.
E.
231.
A. *
B.
C.
D.
E.
232.
A. *
B.
C.
D.
E.
233.
A. *
B.
C.
D.
E.
234.
A. *
B.
C.
tubocurarine, a Nm receptor antagonist, blocks the secretion of catecholamines from adrenal
medulla
trimethaphan, a Nn receptor antagonist, would block the actions of acetylcholine to directly contract
the detrusor muscle of the urinary bladder
acetylcholine activates M receptors at skeletal muscle neuromuscular junction to produce a
contraction
Which of the following is TRUE of fast but not slow response action potentials in cardiac muscle?
phase 0 depolarization is due to Na
resting membrane potentials occur at more positive values for fast than slow response
there is no phase 1
occurs in SA and AV nodal tissue
depolarization can be reduced by Ca channel blockers
Closure of the aortic valve occurs at the onset of which phase of the cardiac cycle?
isovolumetric relaxation
rapid ejection
isovolumetric contraction
atrial systole
rapid ventricular filling
Which of the following events occurs during rapid ejection?
ventricular pressure increases
the T wave of the ECG
mitral valve closes
aortic blood flow decreases
the first heart sound
The second heart sound is produced by:
closure of the aortic valve
opening of semilunar valves
opening of AV valves
closure of AV valves
closure of the mitral valve
Which of the following conditions would increase the afterload placed on the ventricle?
All of the above
Aortic valve stenosis
Increased total peripheral resistance
Emboli
Hypertension
If flow remains constant, the widening of a blood vessel would result in:
an increase in potential energy and a decrease in kinetic energy
an increase in velocity
a decrease in potential energy but no change in kinetic energy
an increase in gravitational energy
an increase in total energy
A doubling of both vessel radius and length would produce which of the following effects on
resistance to flow?
decrease by a factor of 8
decrease by a factor of 16
no change
D.
E.
235.
A. *
B.
C.
D.
E.
236.
A. *
B.
C.
D.
E.
237.
A. *
B.
C.
D.
E.
238.
A. *
B.
C.
D.
E.
239.
A. *
B.
C.
D.
E.
240.
A. *
B.
C.
D.
E.
241.
A. *
B.
increase by a factor of 8
increase by a factor of 16
Which of the following statements is TRUE concerning resistance to flow
The greatest resistance to flow in the cardiovascular system can occur in the arterioles due to their
thick muscle coat.
For vessels arranged in series, the total resistance is determined by summing the reciprocals of the
individual resistance's
An increase in hematocrit would decrease resistance to flow
For vessels arranged in parallel, the individual resistance's are less than the total resistance of the
system.
Flow is directly proportional to resistance
Turbulence:
flow occurs with high velocities of blood flow
occurs in small diameter vessels
occurs with increased production of red blood cells
is determined by the Law of Laplace
requires a lower pressure to maintain
In polycythemia, the increase in red blood cell production will result in which of the following:
Reynold's number will decrease
resistance to flow would decrease
decrease in heart work
flow is increased
viscosity is reduced
Which of the following statements is CORRECT regarding vessel compliance
veins are much more compliant than arteries
arterial compliance increases with age
pulse pressure is not affected by compliance
arterial compliance determines the new level of mean arterial pressure resulting from a change in
cardiac output
lymph vesseis are much more compliant than arteries
An individual with a blood pressure of 140/90:
has a pulse pressure of 50 mm Hg
has a mean arterial pressure of 140 mm Hg
systolic pressure has not been determined
has a diastolic pressure of 140 mm Hg
has a heart rate of 100 beats per minute
All of the following are true concerning mean arterial pressure EXCEPT:
It is dependent on arterial compliance
It is the average pressure in the aorta over time
It is directly proportional to cardiac output
It would be increased with an increase in stroke volume but not change in heart rate or TPR
It is directly proportional to total peripheral resistance
An individual in which heart rate, total peripheral resistance and compliance are kept constant, and
increase in stroke volume results in:
pulse pressure and mean arterial pressure are both increased
no change in pulse pressure or mean arterial pressure
C.
D.
E.
242.
A. *
B.
C.
D.
E.
243.
A. *
B.
C.
D.
E.
244.
A. *
B.
C.
D.
E.
245.
A. *
B.
C.
D.
E.
246.
A. *
B.
C.
D.
E.
247.
A. *
B.
C.
D.
E.
248.
A. *
B.
pulse pressure and mean arterial pressure are both decreased
pulse pressure is increased and mean arterial pressure is decreased
pulse pressure is decreased and mean arterial pressure is increased
Which of the following is a TRUE statement concerning capillaries?
Smooth muscle in arterioles and precapillary sphincters controls flow through capillaries
Continuous capillaries are found in the liver
Fenestrated capillaries allow blood cells to cross the endothelial cells
The endothelial cells are an inert barrier separating blood and tissues
Discontinuous capillaries have narrow gaps between cells
Which of the following statements is TRUE
All of the above
Decreasing radius will reduce wall tension with increasing transmural pressures
Transmural pressure expands the vessel until it is balanced by wall tension
A dilated heart (increased radius), must generate more wall tension to generate a given ventricular
pressure
Larger vessels tend to have larger wall tensions than smaller vessels at any given pressure
Filtration is increased by:
a decrease in plasma oncotic pressure
a decrease in capillary hydrostatic pressure
a decrease in venous pressure
an increase in interstitial hydrostatic pressure
all of the above
Edema will NOT be produced by:
hyperalbuminemia
hypervolemia
histamine
obstruction of lymph vessels
increased venous pressures
Venous return in exercise, would be increased by:
all of the above
increase heart rate
increased respiration
increased muscle activity
increased sympathetic nerve activity
Which statement best describes Autoregulation?
The myogenic hypothesis explains the constant flow seen with arterial pressures between 70 and
180 mm Hg
Active hyperemia can be explained by the Law of Laplace
The decrease in radius seen with an increase in arterial pressure is due to release of norepinephrine
from sympathetic nerves.
Reactive hyperemia could be due to an increase in oxygen levels in blood vessels
Nitric oxide could be involved in the increase in resistance seen with increased arterial pressure
Which of the following agents would result in vasoconstriction?
angiotensin II
histamine
C.
D.
E.
249.
A. *
B.
C.
D.
E.
250.
A. *
B.
C.
D.
E.
251.
A. *
B.
C.
D.
E.
252.
A. *
B.
C.
D.
253.
A. *
B.
C.
D.
E.
254.
A. *
B.
C.
D.
E.
255.
A. *
B.
C.
acetylcholine
bradykinin
Prostacyclin (Prostaglandin I2)
Epinephrine can produce a vasodilation of skeletal muscle blood vessels by activation of which
receptor?
B2 receptors
B1 receptors
B3 receptors
alpha 1 receptors
alpha 2 receptors
An increase in the pressure at the carotid sinus will result in all of the following EXCEPT:
an increase in sympathetic activity to veins
a decrease in cardiac output
vasodilation
a decrease in heart rate
a decrease in total peripheral resistance
Norepinephrine activates which receptor to increase heart rate?
beta 1 receptors
alpha 2 receptors
alpha 1 receptors
beta 2 receptors
beta 3 receptors
A decrease in afterload between heart beats, with constant EDV and contractility, would result in all
of the following in the next beat EXCEPT?
increase in end systolic volume
increase in stroke volume
decrease in peak left ventricular pressure
increase in ejection fraction
Stroke volume can be decreased by:
decreased respiratory rate
lying down with legs raised
decrease in right atrial pressure
a positive inotropic agent
hypotension
A drug is given to your patient that only increases conduction of action potentials through the AV
nodE. You predict that this drug would have which of the following effects on the patient's ECG?
decrease P-R interval
decrease amplitude of T wave
decrease QRS amplitude
shift in mean electrical axis to left
increase P wave duration
Which of the following is an accurate statement about cardiac muscle?
Ca entering the cell triggers release of more Ca from the sarcoplasmic reticulum
Thick and thin filaments are not arranged in sarcomeres
Calmodulin is involved in excitation contraction coupling
D.
E.
256.
A. *
B.
C.
D.
E.
257.
A. *
B.
C.
D.
E.
258.
A. *
B.
C.
D.
E.
259.
A. *
B.
C.
D.
E.
260.
A. *
B.
C.
D.
E.
261.
A. *
B.
C.
D.
262.
A. *
B.
C.
D.
E.
Energy for contraction is derived totally from anaerobic
Contractile force can be varied by the number of muscle fibers activated
A negative inotropic agent would produce which of the following effects on cardiac performance?
decrease the velocity of shortening for any given level of afterload
decrease end diastolic pressure
increase the rate of relaxation of cardiac muscle
increase the force of contraction for any given level of end diastolic volume
increase the rate of rise of ventricular pressure
The gray matter of the spinal cord contains
all of the above
synapses
axons
dendrites
cell bodies of neurons
Which of the following in NOT a characteristic of the parasympathetic system
preganglionic cell bodies are found in the thoracic and lumbar
preganglionic axons are long and postganglionic axons are short
dominates during rest
preganglionic axons exhibit less divergence (innervate fewer postganglionic cells) than those of the
sympathetic system
causes constriction of the pupil
The increase in heart rate and contractility resulting from increased sympathetic activity is mediated
by:
beta 1 receptors
alpha 1 receptors
beta 2 receptors
M 1 receptors
M 2 receptors
A true statement about receptors of the ANS is:
in most, if not all, instances effects of receptor activation are mediated by G-protein activated
processes
NE is a more potent than isoproterenol on beta receptors
the receptors on the target organs of the parasympathetic system are nicotinic
activation of beta receptors reduces cAMP levels
the responses activated through muscarinic receptors are usually faster than those activated through
nicotinic receptors
A change in conductance to which ion is responsible for repolarization of the cardiac action
potential?
K
Na
Ca
Cl
Tissue perfusion is regulated mainly by:
Arterioles
Capillaries
Venules
Aorta
Arteries
263.
A. *
B.
C.
D.
264.
A. *
B.
C.
D.
E.
265.
A. *
B.
C.
D.
E.
266.
A. *
B.
C.
D.
E.
267.
A. *
B.
C.
D.
E.
268.
A. *
B.
C.
D.
E.
269.
A. *
B.
C.
D.
E.
The cardiac cell refractory period is determined by the reactivation of which of the following ion
channels
Na
Cl
K
Ca
Calcium conductance is highest during which phase of the cardiac action potential in ventricular
muscle?
Phase 2
Phase 1
Phase 0
Phase 3
Phase 4
Stimulation of the adrenergic receptors on the pacemaker cells of the heart increases in the membrane
conductance to:
Ca
Na
K
Cl
K and Na
Conduction velocity is the slowest in the heart through the
AV node
Atrial muscle
Bundle of His
Purkinje fibers
Ventricular muscle
Stimulation of the vagus nerve causes which of the following effects on the heart?
increases resting potential ( hyperpolarizes)
increases the slope of the pacemaker potential
increases heart rate
increases AV conduction
increases contractility
Administration of a drug to a patient produces an increase in conduction velocity in the AV node
only. This will produce which of the following effects on this patient's electrocardiogram:
a decrease in PR interval
a decrease in T wave amplitude
a decrease in P wave duration
a decrease in QRS duration
a decrease in QT interval
The precordial leads are:
leads V1 to V6
standard limb leads
augmented limb leads
bipolar leads
standard and augmented limb leads
270.
A. *
B.
C.
D.
E.
271.
A. *
B.
C.
D.
E.
272.
A. *
B.
C.
D.
E.
273.
A. *
B.
C.
D.
E.
274.
A. *
B.
C.
D.
E.
275.
A. *
B.
C.
D.
E.
276.
A. *
B.
C.
D.
E.
277.
A. *
B.
The T wave of the ECG represents
Ventricular repolarization
Atrial repolarization
Ventricular depolarization
Atrial depolarization
Papillary muscle depolarization
Which of the following parts of the ECG represents depolarization of the SA node?
none of the above
QRS complex
PR interval
T wave
P wave
Hypokalemia has all of the following effects EXCEPT
shortens QT interval
reduces K permeability
increased prominence of U wave
flattening of T wave
no answer
In which of the following arrhythmia's would you most likely see a change in the QRS complex?
premature ventricular contractions
junctional premature contractions
atrial premature contractions
atrial fibrillation
first degree heart block
In left bundle branch block:
the QRS complex duration is prolonged
the right and left ventricles are depolarized simultaneously
the T wave is unaffected
the left ventricle is depolarized first
the left atrium is not depolarized
Which of the following DOES NOT produce a positive inotropic response?
Calcium channel blocker
Increase in intracellular [Na]
Norepinephrine
Thyroid hormone
Increased frequency of stimulation
Which of the following variables DOES NOT describe the y-axis of the Starling curve?
ventricular end diastolic volume
ventricular systolic pressure
stroke volume
stroke work
contractile force
A 50% reduction in both tube length and radius will reduce flow by a factor of:
8
4
C.
D.
E.
278.
A. *
B.
C.
D.
279.
A. *
B.
C.
D.
E.
280.
A. *
B.
C.
D.
E.
281.
A.
B. *
C.
D.
E.
282.
A. *
B.
C.
D.
E.
283.
A. *
B.
C.
D.
E.
284.
A. *
B.
C.
D.
2
16
32
Flow is inversely proportional to:
the resistance
the pressure gradient
the radius
the velocity
Turbulence is more likely to occur in a blood vessel if:
the velocity of blood within the vessel is increased
the viscosity of the blood within the vessel is increased
the diameter of the vessel decreases
the density of the blood decreases
the length of the vessel increases
An individual with a blood pressure of 130/73
has a pulse pressure of 57 mm Hg
has a mean arterial pressure of 130 mm Hg
has a systolic pressure of 73 mm Hg
has a diastolic pressure of 92 mm Hg
is suffering from hypotension
All the following statements are true concerning arterial compliance EXCEPT:
arterial compliance with hypotetension decreases
arterial compliance increases with hypertension
arterial compliance decreases with age
arterial compliance is the slope of the aortic static pressure - volume relationship
With an increase in cardiac output, the new mean arterial pressure value is independent of arterial
compliance
An increase in heart rate results in:
a decrease in duration of the refractory period
a decrease in sarcoplasmic Ca levels
an increase in action potential duration
a decrease in K conductance
a decrease in developed force
An increase in mean arterial pressure will result from which of the following:
all of the above
an increase in stroke volume
an increase in cardiac output
an increase in heart rate
none of the above
In an individual with constant cardiac output and total peripheral resistance, an increase in arterial
compliance results in:
a decrease in pulse pressure
an increase in mean arterial pressure
an increase in cardiac work
both a and c
E.
285.
A. *
B.
C.
D.
E.
286.
A. *
B.
C.
D.
E.
287.
A. *
B.
C.
D.
E.
288.
A. *
B.
C.
D.
E.
289.
A. *
B.
C.
D.
E.
290.
A. *
B.
C.
D.
E.
291.
A. *
B.
C.
D.
both b and c
The natural pacemaker of the heart:
has a gradually depolarizing phase 4
is the AV node
is inhibited by fast sodium channel blockers (tetrodotoxin) 4
demonstrates fast response action potentials
is not innervated by the Autonomic nervous system
Incisura on the descending limb of the aortic pressure curve is due to:
closure of the aortic valve
closure of the mitral valve
ventricular systole
atrial systole
atrial repolarization
Which of the following statements BEST describes the Parasympathetic Nervous System?
Preganglionic fibers form the pelvic splanchnic nerves
Preganglionic cell bodies can be found in the intermediolateral cell column at the level of the spinal
cord
It innervates the adrenal medulla
Have long postganglionic and short preganglionic fibers
Preganglionic fibers synapse with postganglionic cell bodies in the paravertebral ganglia
A B2 adrenoceptor antagonist would block which of the following events mediated by
norepinephrine?
vessels, bronchiole relaxation
lipolysis
pupil constriction
constriction of blood
all of the above
Stimulation of the Parasympathetic Nervous System would produce:
contraction of the sphincter muscle of the eye
increased heart rate
ejaculation
relaxation of Gastrointestinal tract smooth muscle
contraction of the bladder internal sphincter
Which of the following statements is TRUE concerning transmitters and receptors of the Autonomic
and Somatic Nervous Systems?
End organs innervated by postganglionic Parasympathetic fibers have M receptors
All postganglionic Sympathetic fibers release acetylcholine
Epinephrine is the major transmitter of the preganglionic Sympathetic fibers
Skeletal muscle has NN receptors on the postsynaptic cell
Acetylcholine activates alpha receptors on all postganglionic cell bodies
Which of the following statements is TRUE concerning the heart valves?
The mitral valve closes when ventricular pressure is greater than atrial pressure
The tricuspid valve is between the left atrium and ventricle
The ventricular pressure required to open the right semilunar valve is greater than that to open the
left semilunar valve
During rapid ejection, both semilunar and atrioventricular valves are open
E.
292.
A. *
B.
C.
D.
E.
293.
A. *
B.
C.
D.
E.
294.
A. *
B.
C.
D.
E.
295.
A. *
B.
C.
D.
E.
296.
A. *
B.
C.
D.
E.
297.
A. *
B.
C.
D.
E.
298.
A. *
B.
C.
D.
E.
During diastole, the semilunar valves are open
Resistance vessels have more ______ than similar sized capacitance vessels
smooth muscle
elastic tissue
endothelial cells
compliance
valves
Inhibition of channel(s) responsible of Ik will produce which of the following effects on the fast
response action potential
both b and c are correct
increase amplitude of plateau phase
increase action potential duration
both a and b are correct
increase amplitude of Phase 0
Stimulation of the Sympathetic nerves to the heart would result in :
a decrease in the interval between the beginning of the P wave to the beginning of the QRS complex
a decrease in the release of Ca from the sarcoplasmic reticulum
a decrease in levels of cAMP
a decrease in ICa through L-type channels
a decrease in If
Post-repolarization refractoriness:
both d and b are correct
causes a ventricular protective effect in atrial fibrillation
is usually seen in ventricular muscle
is due to the slow recovery of Ca channels
all of the above are correct
The duration of ventricular systole is represented on the ECG by:
QT interval
RR interval
PR interval
PP interval
ST interval
Which of the following is correct concerning ECG leads?
Einthoven's Triangle consists of leads I, II, and III
The positive pole of Lead II is at 0o
the hexaxial reference system consists of the precordial and Standard Limb leads
The precordial leads measure in the frontal plane
the augmented leads are part of the transverse plane leads
If a patient has a QRS complex that is positive in lead II and negative in aVR, what is their mean
electrical axis?
between -30 and +120
between -30 and -120
between +150 and -120
between -150 and -60
left axis deviation
299.
A. *
B.
C.
D.
E.
300.
A. *
B.
C.
D.
E.
301.
A. *
B.
C.
D.
E.
302.
A. *
B.
C.
D.
E.
303.
A. *
B.
C.
D.
E.
304.
A. *
B.
C.
D.
E.
305.
A. *
B.
C.
D.
E.
306.
A. *
In sinus arrest:
only b and e are correct
there is a depression of impulse formation in the SA node.
there is an increase in the PR interval
there can be long pauses between P waves
action potentials are blocked as they exit the SA node
Reentry:
only b and c are correct
involves an area of unidirectional block and decremental conduction
can occur in areas that are partially depolarized
only a and b are correct
can occur when two different pathways between two points have homogenous electrophysiological
properties
Which of the following is TRUE of cardiac muscle BUT NOT of skeletal muscle?
The cells are electrically coupled to each other
It contains tropomyosin and troponin
Ca binds to troponin
It is striated muscle.
All of the above
Cardiac contractility is increased by:
inhibition of the Na/K pump
a decrease in the length of the plateau phase of the action potential
a decrease in heart rate
and increase in [Na]o
calcium channel blockers
Which of the following statements about myocardial performance is correct?
only d and c are correct
An increase in afterload will increase stroke volume
A delay between beats will be followed by a larger contraction
An increase in preload will increase peak left ventricular pressure
all of the above are correct
A decrease in afterload before the next cardiac cycle will:
increase velocity of shortening of the left ventricle
increase residual volume in the left ventricle
decrease stroke volume
increase maximal wall tension in the left ventricle
all of the above
Isovolumic relaxation is associated with:
a decrease in left ventricular pressure
the opening of the semilunar valves
the first heart sound
a decrease in left atrial pressure
a decrease in left ventricular volume
In which phase of the cardiac cycle is both the mitral valve open and ventricular pressure falling
rapid ventricular filling
B.
C.
D.
E.
307.
A. *
B.
C.
D.
E.
308.
A. *
B.
C.
D.
E.
309.
A. *
B.
C.
D.
E.
310.
A. *
B.
C.
D.
E.
311.
A. *
B.
C.
D.
E.
312.
A. *
B.
C.
D.
E.
313.
A.
B. *
C.
D.
rapid ejection
isovolumic contraction
atrial systole
reduced ventricular filling
During atrial systole
the mitral valve is open and the aortic valve is closed
the semilunar valves are open and the AV valves are closed
both sets of valves are open
the tricuspid valve is closed and the pulmonic valve is open
both sets of valves are closed
Ejection fraction is:
all of the above
an index of contractility
increased immediately after a decrease in afterload
normally 0.67
comparing volume of blood ejected to volume filling the heart before systole
Which of the following statements concerning Energy is correct?
increase in velocity, will increase the kinetic energy component
Kinetic energy makes up to 10-15% of the total energy on the arterial side of the circulatory system
Total energy is constant along the entire length of the circulatory system
Gravitational energy is increased in vessels above the heart An
In aortic stenosis, the pressure energy component increases
The greatest increase in blood flow would occur with:
a doubling of the vessel radius
a doubling of the pressure gradient and a doubling of vessel length
a halving of the blood viscosity and increasing the pressure gradient by a factor fo 4
a halving of both blood viscosity and vessel length
a doubling of vessel radius and a doubling of blood viscosity
In an aortic aneurysm, there is:
all of the above are correct
an increase in wall stress
an increase in the probability of turbulent flow
only a and b are correct
an increase in pressure energy
The viscosity of blood is decreased by:
All listed
a decrease in tube diameter
an increase in shear rate
anemia
a decrease in hematocrit
Which of the following statements is FALSE concerning compliance?
all of the above
At extremely large volumes, arterial compliance increases
Compliance is the slope of the aortic static pressure - volume curve
arteries are less compliant than veins
E.
314.
A. *
B.
C.
D.
E.
315.
A. *
B.
C.
D.
E.
316.
A. *
B.
C.
D.
E.
317.
A. *
B.
C.
D.
E.
318.
A. *
B.
C.
D.
E.
319.
A. *
B.
C.
D.
E.
320.
A. *
B.
C.
laying down of collagen in arteries decreases compliance
Which of the following statements are correct?
only b and c are correct
pulse pressure increases with a decrease in compliance
pulse pressure and mean arterial pressure can change independently of each other
a change in arterial compliance will result in a change in mean arterial pressure
all of the above are correct
The following information was obtained from a patient while trying to determine the perfect firing
rate for a new implanted pacemaker. When the heart was paced at 60 bpm, Stroke volume = 80 ml
and Cardiac output = 4.8L/min, and when paced at 100 bpm, stoke volume = 48 ml and Cardiac
output = 4.8L/min. Total peripheral resistance was unchanged at both pacing rates. When pacing was
increased from 60 to 100 bpm, you would expect ______ in mean arterial pressure and ______ in
pulse
pressure.
no change;
a decrease
an increase; an increase
an increase; no change
no change; an increase
a decrease; no change
Which of the following statements is correct?
systolic pressure is the pressure attained at the peak of ventricular systole
mean arterial pressure is the difference between the highest and lowest aortic pressures during one
cardiac cycle
the dicrotic notch is due to closing of the mitral valve
diastolic pressure is decreased by an increase in heart rate
pulse pressure is determined by measuring the area under the curve and dividing by the time interval
Which of the following is TRUE concerning capillaries?
discontinuous capillaries allow cells to pass out of them
at rest, all capillaries of a tissue are open
Fenestrated capillaries allow cells to pass out of them
pressures at the venous ends are greater than those at the arterial end of capillaries
precapillary sphincters control blood flow into venules
The Law of Laplace explains:
a, b and c are correct
a small vessel radius reducing wall tension
the greater the wall stress, the greater the pressure generated by the ventricle
only a and b are correct
a thick muscular coat reducing wall stress
Which of the following are correct concerning the driving forces for filtration?
capillary hydrostatic pressure generally decreases along the length of the capillary
capillary hydrostatic pressure is directly proportional to arterial resistance
interstitial fluid hydrostatic pressure is decreased in edema
capillary oncotic pressure is primarily determined by venous pressures
interstitial colloid osmotic pressure is a force that favors absorption
Absorption is decreased by:
all of the above
liver disease
inflammation
D.
E.
321.
A. *
B.
C.
D.
E.
322.
A. *
B.
C.
D.
E.
323.
A. *
B.
C.
D.
E.
324.
A. *
B.
C.
D.
E.
325.
A. *
B.
C.
D.
E.
326.
A. *
B.
C.
D.
E.
327.
A. *
B.
C.
D.
venous obstruction
dilation of arterioles
Which of the following statements BEST describes myogenic control of the peripheral circulation?
with an increase in pressure, the increase wall tension results in a reduction in vessel radius
it is another name for vasomotor tone
with an increase in tissue activity, there is an increase in blood flow mediated via substances
released from active tissues
substances released from endothelial cells control vessel radius
it is extrinsic control of vessel radius
The firing rate of the carotid sinus baroreceptors afferent fibers would be increased by:
a transfusion
a decrease in total peripheral resistance
a decrease in pulse pressure
only a and b are correct
only a and c are correct
A true statement concerning the influence of Sympathetic Nervous System on the circulatory system
is:
Sympathetic stimulation decreases circulating blood volume during exercise Decreasing
Sympathetic activity to the vessels produces a generalized vasodilation
Sympathetic stimulation decreases total peripheral resistance
Sympathetic stimulation decreases circulating blood volume during exercise
Epinephrine activates B2 adrenoceptors producing a vasoconstriction
Sympathetic vasodilator nerves release norepinephrine and produce a vasodilation
A decrease in mean arterial pressure will result in which of the following reflex responses?
all of the above
increased Sympathetic activity to increase end diastolic volume
increased Sympathetic activity to increase total peripheral resistance
increased Sympathetic activity and decreased Parasympathetic activity to increase heart rate
increased Sympathetic activity to increase venous tone
The Bainbridge reflex:
results in increased heart rate and increased urine output
if blocked will results in orthostatic hypotension
predominates in volume depletion
sensors are the chemoreceptors
is activated by Prostaglandins and Bradykinin
Loop ABFH would result from which intervention between cycles?
calcium channel blocker
constriction of veins
administration of an alpha-1 adrenoceptor agonist
hypertension
a decrease in blood volume
Which of the following is TRUE concerning the vascular function curve?
Mean circulatory pressure is the central venous pressure when cardiac output is zero
Increase in central venous pressure will result in an increase in cardiac output
Mean circulatory pressure is independent of blood volume
both a and b are correct
E.
328.
A. *
B.
C.
D.
E.
329.
A. *
B.
C.
D.
E.
330.
A. *
B.
C.
D.
331.
A. *
B.
C.
D.
E.
332.
A. *
B.
C.
D.
E.
333.
A. *
B.
C.
D.
E.
334.
A. *
B.
C.
D.
E.
335.
A. *
all of the above are correct
Metabolic factors are important in the control of:
all of the above are correct
coronary blood flow during stimulation of sympathetic nerves to the heart
cerebral blood flow during exam taking
only b and c are correct
skeletal muscle blood flow during exercise
Net absorption of fluid will occur with all the following EXCEPT
Decreased plasma albumin levels
Constriction of precapillary arterioles
Hemorrhage
Dehydration
Production of bradykinin in inflammation
Capillary hydrostatic pressure is increased by:
increased venous resistance
decrease in arterial pressure
increased arterial resistance
decrease in venous pressure
Which of the following statements is true?
Capillary oncotic pressure is defined by the van Hoff equation
In general, arterial ends are more permeable than venous ends of capillaries
All capillaries have the same permeability
All capillaries have discontinuous endothelium
Endothelium-derived relaxing factor produces an initial vasodilation followed by a long lasting
vasoconstriction
Diffusion will decrease if there is an increase in
Thickness of the membrane
Surface area
Concentration of the molecule in blood compared to interstitial fluid
All of the above
None of the above
Lymph flow is increased by all of the following EXCEPT:
elevated plasma protein concentration
elevated capillary hydrostatic pressure
elevated interstitial fluid protein concentration
histamine
exercise
The venous system can act as a reservoir for peripheral blood chiefly because of the:
large volume capacity of veins
absence of smooth muscle layer in veins
superficial location of veins
low compliance of veins
low O2 saturation in veins
Autoregulation:
Allows tissues to maintain a constant flow by altering arteriolar radius
B.
C.
D.
336.
A. *
B.
C.
D.
E.
337.
A. *
B.
C.
D.
338.
A. *
B.
C.
D.
E.
339.
A. *
B.
C.
D.
E.
340.
A. *
B.
C.
D.
E.
341.
A. *
B.
C.
D.
E.
342.
A. *
B.
C.
D.
Requires functional sympathetic innervation
Explains vessel dilation with elevated pressures
Is controlled by the Florida Department of Motor Vehicles
Which of the following agents will dilate blood vessels?
nitric oxide
serotonin
thromboxane A2
prostaglandin
renin
Epinephrine causes all of the following EXCEPT:
decreases cardiac force of contraction
vasoconstriction
dilates skeletal muscle blood vessels during exercise
increases in heart rate
Occlusion of both carotid arteries between the heart and the carotid sinuses would be expected to
produce:
increase total peripheral resistance
increased activity of baroreceptor afferent nerves
decreased activity of vasomotor center
decrease heart
decreased venous tone
Carotid sinus baroreceptors:
rate of firing is greater with a large pulse pressure
are less sensitive than aortic arch baroreceptors
produce more impulses / sec when mean arterial pressure falls
are more sensitive in chronic hypertension
maximum firing rate occurs at pressures of 50 mm Hg
An increase in mean arterial pressure will produce which of the following effects:
all of the above
decrease in stroke volume by decreasing end-diastolic volume and contractility
decrease total peripheral resistance by decreasing SNS activity to arterioles
decrease in heart rate by an increase in PNS and a decrease in SNS activity to SA node
none of the above
Atrial stretch receptors:
regulates the release of atrial natriuretic peptide
produces changes in heart rate, contractility and vessel resistance
are involved in the Bezold-Jarisch reflex
are activated by an increase in intrathoracic pressure
firing rate is increased by a decrease in venous return
Patients who have received cardiac transplants are able to increase cardiac output during exercise for
which of the following reasons?
increased venous return results in increased stroke volume
ventricular end diastolic pressure is decreased
heart rate is increased by sympathetic stimulation
length of systole is shortened following transplantation
E.
343.
A. *
B.
C.
D.
E.
344.
A. *
B.
C.
D.
E.
345.
A. *
B.
C.
D.
E.
346.
A. *
B.
C.
D.
E.
347.
A. *
B.
C.
D.
E.
348.
A. *
B.
C.
D.
E.
349.
A. *
B.
C.
D.
E.
350.
none of the above
Which of the following decreases contractility?
severe hypoxia
glucagon
adrenal corticoid hormones
insulin
thyroid hormone
Which of the following events is NOT a consequence of an increase in sympathetic nerve activity?
decreased AV conduction
decreased duration of systole
decreased contraction duration
increased peak ventricular pressure
increased mean arterial pressure
Coronary blood flow is increased by all of the following EXCEPT:
increased myocardial wall tension
adenosine
increased metabolic activity
cardiac sympathetic nerve stimulation
increased aortic pressure
Anaphylaxis is an example of
vasogenic shock
cardiogenic shock
obstructive shock
hypovolemic shock
electric shock
Activation of the baroreceptor reflex in hemorrhage results in vasoconstriction in all of the following
vascular beds EXCEPT:
cerebral
skeletal
renal
cutaneous
GI
Which of the following is a decompensatory mechanism in hemorrhage?
acidosis
chemoreceptor reflex
cerebral ischemia
release of vasoconstrictor agents
fluid retention
Orthostatic hypotension can result from all the following interventions EXCEPT:
a nicotinic ganglionic receptor antagonist
b-adrenergic receptor antagonists
inhibition of sympathetic nerve function
a-adrenergic receptor antagonists
inhibition of dopamine b-hydroxylase
During exercise, blood flow to skeletal muscles increases because of which of the following factors?
A. *
B.
C.
D.
E.
351.
A. *
B.
C.
D.
E.
352.
A. *
B.
C.
D.
E.
353.
A. *
B.
C.
D.
E.
354.
A. *
B.
C.
D.
E.
355.
A. *
B.
C.
D.
E.
356.
A. *
B.
C.
D.
E.
circulating epinephrine dilates skeletal muscle blood vessels
activity of parasympathetic nerve fibers to the heart increases
resistance of the veins and venules decreases
total peripheral resistance increases
all of the above
All of the following will help the heart to compensate for the reduction in contractility that occurs
during heart failure EXCEPT an increase in:
total peripheral resistance
ventricular wall thickness
sympathetic nerve discharge
end-diastolic volume
retention of fluid by the kidney
Which of the following statements about exercise is INCORRECT?
The myogenic hypothesis accounts for the vascular effects seen in skeletal muscle during exercise
Blood volume actually declines due to fluid losses
The increase in mean arterial pressure is due to an increase in cardiac output
the vasodilation in active muscles causes the decrease in total peripheral resistance seen in exercise.
Changes in arterial pressure parallel the severity of exercise
In an upright posture, which of the following is the highest?
transmural pressure in the femoral artery at the level of the ankle
mean arterial pressure in the ascending aorta
mean arterial pressure in the carotid sinus
central venous pressure
peak ventricular pressure during systole
Reactive hyperemia is mediated by:
metabolic factors
elevation of parasympathetic activity
reflex increase in sympathetic mediated vascular resistance
Myogenic hypothesis
intrinsic stretch of vascular smooth muscle
Mean circulatory pressure is increased by:
increased blood volume
decreased arterial resistance
increased arterial resistance
decreased blood volume
all of the above
With very severe exercise, you would expect which of the following to occur?
stroke volume reaches a maximum and may decrease
heart rate continues to rise
decrease in CO2 levels
pH usually increases
decrease in lactic acid production